Download as pdf or txt
Download as pdf or txt
You are on page 1of 119

GMATCLUBFLASHCARDS

http://gmatclub.com/flashcards

ContributingtoEachOthersLearning
GMAT Club 2011 1
GMAT Club Marketplace
Find this and other great free products are available
for download from the GMAT Club Marketplace.
Find the best free resources and also the best
deals to save the most money on books, tests,
courses, iPhone Apps, and admissions consulting.

http://gmatclub.com/marketplace

GMAT Club 2011 2

GMAT Club contributing to each others learning


GMAT MATH

GMAT Club 2011 3


Percent Percent 2
1 3

What is the percent formula? What is the formula for percent change?

If Jack got a raise from $15 per hour to $18


per hour, what was the percent increase?
How much is 15 percent of 20?

GMAT Club contributing to each others learning MATH: ARITHMETIC GMAT Club contributing to each others learning MATH: ARITHMETIC

Percent | Answers Percent 2 | Answers


2 4

Percent Formula: Percent Change Formula:


Part = Percent x Whole
100%

For example, 15% of 20 is:
15 300 Jacks Raise:
20 = =3
100 100 18 15 3 100
100% = 100% = = 20%
15 15 5
! Make sure you use the original amount (15), not
the new one (18)

GMAT Club contributing to each others learning MATH: ARITHMETIC GMAT Club contributing to each others learning MATH: ARITHMETIC

GMAT Club 2011 4


Percent 3 Percent 4
5 7

If the production of hybrid cars tripled last year, by


how many percent did it increase?
100%

200% 50% of 25 is 25% of which number?


250%

300%

333%

GMAT Club contributing to each others learning MATH: ARITHMETIC GMAT Club contributing to each others learning MATH: ARITHMETIC

Percent 3 | Answers Percent 4 | Answers


6 8

The correct answer is B increased by 200% There are 2 solutions to this problem:
For example, if production was 10 cars, and it Long one:
tripled to 30 cars, the increase was 20 cars, which 50% of 25 is 12.5
is 200% of 10 12.5 *100 = x * 25
x = 50

Short one but a harder to come up with:


50% * 25 = x * 25%
50 = x

GMAT Club contributing to each others learning MATH: ARITHMETIC GMAT Club contributing to each others learning MATH: ARITHMETIC

GMAT Club 2011 5


Odd/Even Rules Odd/Even Rules 2
9 11

Check your knowledge of Odd/Even rules


Odd + Odd = ?

Odd Even = ?

Even + Odd = ? Is 0 odd or even?


Odd x Odd = ?

Odd Even = ?

Odd x Even = ?

Hint: try picking numbers

GMAT Club contributing to each others learning MATH: ARITHMETIC GMAT Club contributing to each others learning MATH: ARITHMETIC

Odd/Even Rules | Answers Odd/Even Rules 2 | Answers


10 12

Odd + Odd = 1+ 1 = 2 (Even)


Odd Even = 3 2 = 1 (Odd)
Even + Odd = 2 + 1 = 3 (Odd)
Odd x Odd = 3 x 3 = 9 (Odd) 0 (zero) is Even
Odd Even = 3 2 = 1.5 (Not an integer!)
It is also not positive or negative
Odd x Even = 3 x 2 = 6 (Even)
it is neutral

GMAT Club contributing to each others learning MATH: ARITHMETIC GMAT Club contributing to each others learning MATH: ARITHMETIC

GMAT Club 2011 6


Odd/Even Rules 3 Odd/Even Rules 4 (Ultra Hard)
13 15

If b is an odd number, which of the following Is A+B+C even or odd?


must be even?
A. 2 3 A C B is even
;1 ;
B.
2 is odd


C. 1
2
D. 2 + 2
E. 2 + 2

GMAT Club contributing to each others learning MATH: ARITHMETIC GMAT Club contributing to each others learning MATH: ARITHMETIC

Odd/Even Rules 3 | Answers Odd/Even Rules 4 | Answers


14 16

Use the plug numbers method to check each Statements (1) and (2) combined are insufficient.
answer choice. Lets take 1: Consider A=6, B=4, and C=2 (the answer is "yes")
A. 2*1 3 = -1 (Odd) and A=0.5, B=0.3, and C=0.2 (the answer is "no").
1;1
B. = 0 (Even) however, just in case, lets try 3. Do not assume that the numbers are integers if
2
3;1 the question does not mention it.
= 1 (Odd). It usually is a good idea to run
2
through 2 different numbers if you get zero or similar The correct answer is E.
1 1
C. 1 = (not an intenger)
2 2
D. 1 + 2 = 3 (Odd)
E. 2 + 2 = 4 (Even)

GMAT Club contributing to each others learning MATH: ARITHMETIC GMAT Club contributing to each others learning MATH: ARITHMETIC

GMAT Club 2011 7


Divisibility Rules Divisibility 2 (Hard)
17 19

Is 54780 divisible by 2? Is integer 2 4 divisible by 9?


Is 1671 divisible by 3? x is an integer divisible by 3
Is 5632 divisible by 4? xy is an integer divisible by 9
Is 3830 divisible by 5?
Is 2658 divisible by 6?
Is 396 divisible by 9?

GMAT Club contributing to each others learning MATH: ARITHMETIC GMAT Club contributing to each others learning MATH: ARITHMETIC

Divisibility Rules | Answers Divisibility 2 | Answers


18 20

Yes. Divisibility by 2 last digit of a number is even The best way to approach this question is to plug
Yes. Divisibility by 3 sum of all digits is a multiple in several sets of numbers
of 3 Many are tempted to plug 3 for x and then for S2,
Yes. Divisibility by 4 last 2 digits is a multiple of 4 the only value y can have is 3; in that case, the
Yes. Divisibility by 5 the last digit is either 0 or 5 answer is Yes.
1
Yes. Divisibility by 6 the sum of digits is a multiple But if we try x=81 and y=9, then x is an integer
of 3 and the last digit is even divisible by 3, xy is an integer divisible by 9, but
Yes. Divisibility by 9 the sum of digits is a multiple 2 4 = 1 and is not divisible by 9.
of 9 The answer is E

GMAT Club contributing to each others learning MATH: ARITHMETIC GMAT Club contributing to each others learning MATH: ARITHMETIC

GMAT Club 2011 8


Number Properties Number Properties 2 (Hard)
21 23

Which of the following integers represents a sum If 2 = + 5, = 2, =


of 3 consecutive even integers?
2, 3 + 2 + 7?
200

303
x>0
400 y=4
554

570

GMAT Club contributing to each others learning MATH: ARITHMETIC GMAT Club contributing to each others learning MATH: ARITHMETIC

Number Properties | Answers Number Properties 2 | Answers


22 24

To answer the question, check which integer is Substitute x, y, and z in the original equation to get the
both divisible by 3 (since there are 3 integers) and following: 2 2 3 = 0; The solutions to this
equation are x=-1 and x=3.
is even. The only number that falls into both of
Statement 1 is sufficient. If x>0, then the only solution
those 2 categories is 570. is x=3. The result is 27+16+6 =49, which is divisible by
Correct answer is E 7
Statement 2 is sufficient. If y=4, plugging in this value
into 2 = + 5, gives us that x=3 or x=-3. However,
based on the above, x=3 or x=-1. Therefore x=3.
Sufficient.
The correct answer is D.

GMAT Club contributing to each others learning MATH: ARITHMETIC GMAT Club contributing to each others learning MATH: ARITHMETIC

GMAT Club 2011 9


Number Properties 3 Multiples
25 27

How many distinct integers are What is a multiple?


there between 1 and 21 inclusive?
Is 5 a multiple of 55 or is 55 a
multiple of 5?

GMAT Club contributing to each others learning MATH: ARITHMETIC GMAT Club contributing to each others learning MATH: ARITHMETIC

Number Properties 3 | Answers Multiples | Answers


26 28

Most often GMAT questions will say inclusive but The multiple of a number is the
sometimes they dont need to remember to
check product of the number and any
The formula for calculating the number of integers other whole number. (For example,
between two numbers is: N-M+1
Therefore, the answer is: 21 1 + 1 = 21 2,4,6,8 are multiples of 2)
You can also write out all of the numbers though it Therefore 55 is a multiple of 5
is not always possible but just in case:
1, 2, 3, 4, 5, 6, 7, 8, 9, 10, 11, 12, 13, 14, 15, 16, 17, For example, 6 has factors 1, 2, 3, 6;
18, 19, 20, 21 and multiples that are 6, 12, 24, 36
GMAT Club contributing to each others learning MATH: ARITHMETIC GMAT Club contributing to each others learning MATH: ARITHMETIC

GMAT Club 2011 10


Multiples 2 Multiples 3
29 31

Is 0 (zero) a multiple of 100? How to find a Least Common


Multiple (LCM)?

GMAT Club contributing to each others learning MATH: ARITHMETIC GMAT Club contributing to each others learning MATH: ARITHMETIC

Multiples 2 | Answers Multiples 3 | Answers


30 32

Option 1:
Step 1: Find the prime factors of each of the numbers
Yes, 0 (zero) is a multiple of

Step 2: Multiply the unique factors (exclude


everything duplicates)
However, it is unlikely that GMAT Option 2:
Step 1: Multiply the two numbers
will test this property but it helps to Step 2: Find any factors the two numbers share
remember Step 3: Divide the product in Step 1 by the factors
that the two numbers have in common

GMAT Club contributing to each others learning MATH: ARITHMETIC GMAT Club contributing to each others learning MATH: ARITHMETIC

GMAT Club 2011 11


Multiples 4 Translate
33 35

The product of three and four is reduced by


What is the Least Common five and then increased by the difference
between the original product and eight = ?
Multiple of 18 and 24?

GMAT Club contributing to each others learning MATH: ARITHMETIC GMAT Club contributing to each others learning MATH: ARITHMETIC

Multiples 2 Answers Translate Answers


34 36

Option 1
Find the factors of 18 and 24: 3 x 4 = 12
18 = 3 x 3 x 2
12 5 = 7
24 = 3 x 2 x 2 x 2
Multiply the unique factors: 3 x 3 x 2 x 2 x 2 = 72
12 8 = 4
Option 2 7 + 4 = 11
Multiply the 2 numbers: 18 x 24 = 432 Answer: 11
Shared factors: 2 and 3
Divide 432 by 2 and 3 = 72

GMAT Club contributing to each others learning MATH: ARITHMETIC GMAT Club contributing to each others learning MATH: ARITHMETIC

GMAT Club 2011 12


Arithmetic to Memorize Arithmetic to Memorize 2
37 39

1 1
= =
2 12
1 7
= =
8
4 3
2 =
= 4
5
1
75% =
= 20% =
20
1 2
= 16 % =
8 3
1 1
83 % =
= 3
6
GMAT Club contributing to each others learning MATH: ARITHMETIC GMAT Club contributing to each others learning MATH: ARITHMETIC

Arithmetic to Memorize | Answers Arithmetic to Memorize 2 | Answers


38 40

1 1
= 50% = 8.33%
2 12
1 7
= 25% = 87.5%
8
4 3
2 = 75%
= 40% 4
5
1
75% =
= 5% 1
20 20% =
5
1 2 1
= 12.5% 16 % =
8 3 6
1 1 5
= 16.67% 83 % =
6 3 6

GMAT Club contributing to each others learning MATH: ARITHMETIC GMAT Club contributing to each others learning MATH: ARITHMETIC

GMAT Club 2011 13


Arithmetic to Memorize 3 Arithmetic to Memorize 4
41 43

2
2 = 32 =
3
2 = 33 =
24 = 34 =
25 = 35 =
26 = 42 =
27 = 43 =
44 =

GMAT Club contributing to each others learning MATH: ARITHMETIC GMAT Club contributing to each others learning MATH: ARITHMETIC

Arithmetic to Memorize 3 | Answers Arithmetic to Memorize 4 | Answers


42 44

2
2 =4 32 = 9
3
2 =8 33 = 27

4
2 = 16 34 = 81
25 = 32 35 = 243
26 = 64 42 = 16
27 = 128 43 = 64
44 = 256

GMAT Club contributing to each others learning MATH: ARITHMETIC GMAT Club contributing to each others learning MATH: ARITHMETIC

GMAT Club 2011 14


Arithmetic to Memorize 5 Arithmetic to Memorize 6
45 47

5
2 =
5
3 = Complete the following:
112 = 8, 16, 24, 32, 40, 160
122 =
132 = 12, 24, 36. 120
142 =
152 = 15, 30 150
162 =

GMAT Club contributing to each others learning MATH: ARITHMETIC GMAT Club contributing to each others learning MATH: ARITHMETIC

Arithmetic to Memorize 5 | Answers Arithmetic to Memorize 6 | Answers


46 48

5
2 = 25
5
3 = 125 Complete the following (Answers):

2
11 = 121 8, 16, 24, 32, 40, 48, 56, 64, 72, 80, 88, 96, 104,
122 = 144 112, 120
132 = 169
142 = 196 12, 24, 36, 48, 60, 72, 84, 96, 108, 120
152 = 225
162 = 256 15, 30, 45, 60, 75, 90, 105, 120

GMAT Club contributing to each others learning MATH: ARITHMETIC GMAT Club contributing to each others learning MATH: ARITHMETIC

GMAT Club 2011 15


Arithmetic to Memorize 7 Reciprocal
49 51

2=
3=
625 =
169 = What is a reciprocal?
List all Primes between 1 and 50
Extra Hard & Extra Credit
6
2 =
3
125 =
5
243 =
GMAT Club contributing to each others learning MATH: ARITHMETIC GMAT Club contributing to each others learning MATH: ARITHMETIC

Arithmetic to Memorize 7 | Answers Reciprocal Answers


50 52

2 = 1.4 1
Reciprocal for a number , denoted by or
3 = 1.7 ;1 , is a number which when multiplied by
625 = 25
yields 1. The reciprocal of a fraction is .
169 = 13 1
Primes: 2, 3, 5, 7, 11, 13, 17, 19, 23, 29, 31, 37, 41, 43, For example reciprocal of 3 is 3
47 5 6
6
Reciprocal of 6 is 5.
2 = 64
3
125 = 5
5
243 = 3
GMAT Club contributing to each others learning MATH: ARITHMETIC GMAT Club contributing to each others learning MATH: ARITHMETIC

GMAT Club 2011 16


Absolute Value Absolute Value 2
53 55

What is the 3- step approach to solving If |x-1| = 4, what is the value of


equations and inequalities with absolute x?
value?

GMAT Club contributing to each others learning MATH: ARITHMETIC GMAT Club contributing to each others learning MATH: ARITHMETIC

Absolute Value | Answers Absolute Value 2 | Answers


54 56

Step1: Open modulus and set conditions. Step 1: Find positive/negative roots:
To solve/open a modulus, you need to 1 0 in our case, 1 = 4
consider 2 situations to find all roots: 1 < 0 in our case, 1 = 4
Positive(or rather non-negative) Step 2: Solve the equations
x =5
Negative
x = -3
Step 2: Solve new equations from Step 1
Step 3: Check conditions ( 1 0 & 1 < 0)
Step 3: Check conditions for each
5 1=40
solution from Step 2 -3 1 = -4 < 0

GMAT Club contributing to each others learning MATH: ARITHMETIC GMAT Club contributing to each others learning MATH: ARITHMETIC

GMAT Club 2011 17


Absolute Value 3 Absolute Value 4
57 59

Is M<0?
= ||
2
What is the value of x If |x 5| = 10? =9

GMAT Club contributing to each others learning MATH: ARITHMETIC GMAT Club contributing to each others learning MATH: ARITHMETIC

Absolute Value 3 | Answers Absolute Value 4 | Answers


58 60

|x 5| = 10 Statement (1) by itself is not sufficient. From


We must evaluate both the positive and negative statement (1), M is either a negative number or
outcome since the absolute value removes the zero. If M=-3, then (-3)=|-3| or 3=3, which is not
negative sign sufficient.
Thus, we have 2 equations: Statement (2) by itself is not sufficient. From
X 5 = 10
statement (2), M can be either 3 or -3, which is not
sufficient.
X 5 = - 10 (in case it was a negative expression)
Thus x = 15 and -5. (Plug in both to check) Statements (1) and (2) combined are sufficient. If
we combine both statements, then M=-3.
Absolute value inequalities and equations will almost
always have 2 answers/solutions! The correct answer is C.

GMAT Club contributing to each others learning MATH: ARITHMETIC GMAT Club contributing to each others learning MATH: ARITHMETIC

GMAT Club 2011 18


Factors Factors 2
61 63

How many total factors does 462 Do integers usually have an odd or
have? even number of factors?

GMAT Club contributing to each others learning MATH: ARITHMETIC GMAT Club contributing to each others learning MATH: ARITHMETIC

Factors | Answers Factors 2 | Answers


62 64

Advanced method for finding the number of all possible Vast majority of integers have an even number of
factors:
factors such as 12 for example: 1, 2, 3, 4, 6, 12.
Write out each prime factor and their power, for
example, 12 can be written as follows: Thus when finding the total number of factors,
22
make sure it is an even number
31 The only integers that have an odd number of
50 etc (all other primes will have a zero power) factors are perfect squares (thats numbers such as
Add 1 to all powers and multiply them: (in this case 6) 4, 9, 16, 25, 36, etc). Try 25 for example, it has
and thats the number of factors (1, 2, 3, 4, 6, 12)
1 1 1 1
factors of 1, 5, and 25. Factors of 36 are 1, 2, 3, 4,
Prime Factors of 462 = 2 , 3 , 7 , 11
4
6, 9, 12, 18, 36.
Total number of factors: 2x2x2x2 = 2 = 16

GMAT Club contributing to each others learning MATH: ARITHMETIC GMAT Club contributing to each others learning MATH: ARITHMETIC

GMAT Club 2011 19


Ratios Powers
65 67

What is the ratio of a to d if


1
2a=3b, =2c, and 3c=d? 22 + 23 + 24 + 2 = ?
2 A) 29 + 2
B) 210
A)1:2 B)2:1 C)2:3 D)4:3 E)1:5 C) 25 + 2
D) 25
E) 30
GMAT Club contributing to each others learning MATH: ARITHMETIC GMAT Club contributing to each others learning MATH: ARITHMETIC

Ratios Powers | Answers


66 68

With questions like these, unless you can spot a


shortcut right away, the easiest way to solve is to
plug numbers: lets pick a=6, since there is a 2 and
3 involved 22 + 23 + 24 + 2 = ?
2*6 = 3b; b = 4 There are no rules for adding or

0.5*4 = 2c; c = 1 subtracting powers. This question is


3*1 = d; d = 3 solved by brute force:
Therefore, the ration of a to d is 2:1 4 + 8 + 16 + 2 = 30
The correct answer is B. The correct answer is (E)

GMAT Club contributing to each others learning MATH: ARITHMETIC GMAT Club contributing to each others learning MATH: ARITHMETIC

GMAT Club 2011 20


Powers 2 Powers 3
69 71

22 23 24 25 2 = ?
A) 2120 24 28
=?
B) 3215 26 2
C) 215
D) 214
E) 62

GMAT Club contributing to each others learning MATH: ARITHMETIC GMAT Club contributing to each others learning MATH: ARITHMETIC

Powers 2 | Answers Powers 3 | Answers


70 72

2 3 4 5 15
2 2 2 2 2= 2
When powers with the same base (2 24 28 212
6 = = 25
in this case) are multiplied, the powers 2 2 27
are summed and the base is held Division of powers with the same base
constant. Thus, it is 2 to the power of (2) is handled similarly to multiplication
2+3+4+5+1=15. except the power values are deducted.
The base stays unchanged
The correct answer is (C) 215
GMAT Club contributing to each others learning MATH: ARITHMETIC GMAT Club contributing to each others learning MATH: ARITHMETIC

GMAT Club 2011 21


Powers 4 Powers 5
73 75

(33 )2 = ?
3
33 = ?

GMAT Club contributing to each others learning MATH: ARITHMETIC GMAT Club contributing to each others learning MATH: ARITHMETIC

Powers 4 | Answers Powers 5 | Answers


74 76

(33 )2 = 36
3
When a number taken to a power is 33 = 327
taken to another power, you need to Start operations from outside and work your way
in: 3 taken to the power of 3 is 27. Thus the base of
multiply the exponents (2 * 3) 3 is taken to the power of 27.

GMAT Club contributing to each others learning MATH: ARITHMETIC GMAT Club contributing to each others learning MATH: ARITHMETIC

GMAT Club 2011 22


Powers 6 Powers 7
77 79

2;2 2;3 26 2;4 =?


50 = ?

GMAT Club contributing to each others learning MATH: ARITHMETIC GMAT Club contributing to each others learning MATH: ARITHMETIC

Powers 6 | Answers Powers 7 | Answers


78 80

2;2 2;3 26 2;4 = 2;3 =


1
=
1
50 = 1
23 8
Any number (positive or
Negative powers turn a number to a
reciprocal number taken to that power. negative) taken to the power of 0
1
For example 2;2 = 2 =
1 (zero) equal to 1.
2 4
Not tested on the GMAT, zero to
the power of zero is also 1.
GMAT Club contributing to each others learning MATH: ARITHMETIC GMAT Club contributing to each others learning MATH: ARITHMETIC

GMAT Club 2011 23


Powers 8 Powers 9 (Ultra Hard)
81 83

If m and n are positive integers, is the remainder of


10 : 10 :
3
larger than the remainder of 3
?
1
m>n
4 =?

2
The remainder of is 2
3

GMAT Club contributing to each others learning MATH: ARITHMETIC GMAT Club contributing to each others learning MATH: ARITHMETIC

Powers 8 | Answers Powers 9 | Answers


82 84

Statement (1) by itself is insufficient. In expression


10 + , the sum of digits of 10 is always 1 and it is
10 :
the value of n that determines the remainder of
3
1 . If you plug in m=2, n=1 (the answer is "yes") and m=3,
4 = 4=2
2 n=2 (the answer is "no").
Statement (2) by itself is sufficient. If the remainder of

Fraction powers are interpreted as 3
is 2, as S2 states, then is 2, 5, or 8 and the sum of the
10 :
follows: the denominator is the root and digits of 3 is divisible by 3. Therefore, the
10 :
numerator is the power. For example remainder of 3 is 0, which cannot be larger that
2 1 10 :
3 2 the remainder of no matter what m is.
3 = 3 3 or 2 = 2 2 The correct answer is B.
3

GMAT Club contributing to each others learning MATH: ARITHMETIC GMAT Club contributing to each others learning MATH: ARITHMETIC

GMAT Club 2011 24


Percentiles ConsecutiveNumbers
85 87

Lenasfirsttestscorewasatthe80th percentilein
aclassof120students.Onanothertest,24outof Whatisthesumofconsecutiveintegers
200studentsscoredbetterthanLena.Ifnobody
hadLenasscore,whatisLenaspercentileafter 9,8,7,6,5,4,3,2,1,0,1,2?
thetwotests?

GMATClub contributingtoeachotherslearning MATH:ARITHMETIC GMATClub contributingtoeachotherslearning MATH:ARITHMETIC

Percentiles|Answers ConsecutiveNumbers|Answers
86 88

Takeeachtestresultseparately: Sumofconsecutiveintegersequalsthemean
Test1:120x80th=96(shescoredbetterthan96 multipliedbythenumberofterms,n.Given
students) consecutiveintegers9,8,7,6,5,4,3,2,1,0,1,2
Test2:Shescoredbetterthan176students(200 24) 3.5 (meanequalstothe
Sumuptheresults: averageofthefirstandlastterms),sothesum
96+176=272 equalsto 3.5 12 42 .
120+200=320

85th percentile

GMATClub contributingtoeachotherslearning MATH:ARITHMETIC GMATClub contributingtoeachotherslearning MATH:ARITHMETIC

GMAT Club 2011 25


Evenly Spaced Set Recurring Decimal
89 91

What is the sum of all members of Express 0.393939 in a fraction format


the set 9,12,15,18,21,24 ?

GMAT Club contributing to each others learning MATH: ARITHMETIC GMAT Club contributing to each others learning MATH: ARITHMETIC

Evenly Spaced Set | Answers Recurring Decimal | Answers


90 92

The sum of elements of evenly spaced set is given To convert a recurring decimal to fraction:
by the formula 1. Separate the recurring number from the decimal
1 :
fraction
Sum = 2. Annex denominator with "9" as many times as the
2
9:24
length of the recurring number
Therefore, 6 = 99 3. Reduce the fraction to its lowest terms
2

Example #1: Convert 0.3939 to a fraction


1: The recurring number is 39
39
2: - the number is 2 digits so two nines are added
99
13
3: Reducing it to lowest terms:
33

GMAT Club contributing to each others learning MATH: ARITHMETIC GMAT Club contributing to each others learning MATH: ARITHMETIC

GMAT Club 2011 26


Fractions (Extra Hard)
93

If a, b, and c are positive distinct



( )

integers, is an integer?

c=2
a=b+c

GMAT Club contributing to each others learning MATH: ARITHMETIC

Fractions | Answers
94

Statement I is insufficient since it does not provide


enough information about b or c

( )

Statement II: We can rewrite as

: 1 1
Now plug in the value for a from S2:
=+
since b and c are disticnt positive integers and b is
not equal to c, the expression cannot be an integer
The correct answer is B

GMAT Club contributing to each others learning MATH: ARITHMETIC

GMAT Club 2011 27


Algebra(Hard)
1

Ifeachexpressionunderthesquarerootisgreater
thanorequalto0,whatis 6 9
2 3?
2
2 6 2
2 3
2 6 2
2

GMATClub contributingtoeachotherslearning MATH:ALGEBRA

Algebra|Answers
2

Basedonthesetup(andMathprinciplestestedon
theGMAT), 2 hastobe 0.Therefore
2.
6 9 3 3
Because 2, 3 0
Therefore:3 2 3 2
ThecorrectanswerisA

GMATClub contributingtoeachotherslearning MATH:ALGEBRA

GMAT Club 2011 28


Average Average2
1 3

Iftheaverageof5consecutiveintegersis Theaverageof11co nsecutiveintegersis12.


12,whatistheaverageoftheevenonly Then,9isdeductedfromthefirstconsecutive
integers? number,8isdeductedfromthesecond,7is
deductedfromthethird,andsoonuntilthe
10
lastnumberwhichremainsunchanged.What
12
isthenewaverage?
13.5
18
A)55 B)50 C)6 D)7.5 E)8.7
36

GMATClub contributingtoeachotherslearning MATH:STATISTICS GMATClub contributingtoeachotherslearning MATH:STATISTICS

Average|Answers Average2|Answers
2 4

First,findtheconsecutiveintegers.Since Youdontneedtofindeachofthenumbers.
thereare5,theremaybeeither2or3even Instead,youhavetwooptions,youcandeductthe
integers.Theseintegersare10,11,12,13,14. averageofnumbersbetween0and9(thereisa
Theaverageoftheevenintegersis12aswell trapthough,thereshouldbe10numbersrather
than9andtheaverageis4.5,not5)oryoucan
(10+12+14=36.Divide36by3andyouwill
findthesumofconsecutiveintegers(11*12=132)
get12). andsubtractthesumofintegersfrom0to9(45)
ThecorrectanswerisB. anddividetheresult(87) by 10.
ThecorrectanswerisE.8.7

GMATClub contributingtoeachotherslearning MATH:STATISTICS GMATClub contributingtoeachotherslearning MATH:STATISTICS

GMAT Club 2011 29


Mean Median
5 7

HowtofindtheMean?
HowtofindtheMedian?

GMATClub contributingtoeachotherslearning MATH:STATISTICS GMATClub contributingtoeachotherslearning MATH:STATISTICS

Mean Median|Answers
6 8

Arrangeallnumbersinanorder
ArithmeticMean=Average= fromthesmallesttothelargest.The
Median willbethemiddlenumber.
number of elements Ifthereisanevennumberof
elements,theMedian willbethe
averageofthe2middlenumbers

GMATClub contributingtoeachotherslearning MATH:STATISTICS GMATClub contributingtoeachotherslearning MATH:STATISTICS

GMAT Club 2011 30


Mode Range
9 11

Howtofindthemode? HowtofindtheRange?

GMATClub contributingtoeachotherslearning MATH:STATISTICS GMATClub contributingtoeachotherslearning MATH:STATISTICS

Mode|Answers Range|Answers
10 12

TheModeofanarrayisthenumberthat Rangeisthedifferencebetweenthe
appearsmostoften.Forexample,inan smallestandlargestelementsofan
array1,2,3,3,4 theMode is3.It array.Ifyouhavetofindmorethanjust
appearedtwice.Inthearray1,2,3,3,4,4 range,itisalwaysagoodideatorewrite
theMode is3and4(therecanbemore theelementsinancreasingorder
thanonemode).

GMATClub contributingtoeachotherslearning MATH:STATISTICS GMATClub contributingtoeachotherslearning MATH:STATISTICS

GMAT Club 2011 31


StandardDeviation StandardDeviation2
13 15

Whichofthesetshasahigherstandarddeviation?
SetA SetB
HowtofindtheStandard

Deviation?

GMATClub contributingtoeachotherslearning MATH:STATISTICS GMATClub contributingtoeachotherslearning MATH:STATISTICS

StandardDeviation|Answers StandardDeviation2|Answers
14 16

YouwonthavetocalculateSDontheGMATbut SetAhasthehigherStandardDeviationbecause
youneedtounderstandtheconceptofSD theelementsaredistributedfurtherfromthe
StandardDeviationmeasureshowspreadoutthe mean
membersofthearrayare.TofindtheStandard
Deviation:
Findthemean
Findthedifferencebetweeneachnumberandthe
mean
Squareeachofthedifferences
Findtheaverageofthesquareddifferences
Takeasquarerootoftheaverage

GMATClub contributingtoeachotherslearning MATH:STATISTICS GMATClub contributingtoeachotherslearning MATH:STATISTICS

GMAT Club 2011 32


StandardDeviation3 StandardDeviation4
17 19

Whatisthefastestwaytoestimatestandard WhatistheStandardDeviationofaset
deviation(withoutcalculatingit)?
ofconsecutiveevenintegers?
(1)Thereare39elementsintheset.
Thereisaset{67,32,76,35,101,45,24,37}.Ifwe
createanewsetthatconsistsofallelementsof
(2)themeanofthesetis382
theinitialsetbutdecreasedby17%,whatisthe
changeinstandarddeviation?

GMATClub contributingtoeachotherslearning MATH:STATISTICS GMATClub contributingtoeachotherslearning MATH:STATISTICS

StandardDeviation3|Answers StandardDeviation4|Answers
18 20

Wedon'tneedtocalculateasdecreaseinall BeforereadingDataSufficiencystatements,whatcan
elementsofasetbyaconstantpercentagewill wesayaboutthequestion?Whatshouldweknowto
decreasethestandarddeviationofthesetbythe findstandarddeviation?"consecutiveevenintegers"
samepercentage(theaverageisdecreasedby17% meansthatallelementsstrictlyrelatedtoeachother.
aswellasthedifferencebetweenaverage(mean) Ifweshiftthesetbyaddingorsubtractinganyinteger,
itdoesnotchangethestandarddeviation.Onething
andallelementsortheirsquares.Thusthe
weshouldknowisthenumberofelementsintheset,
decreaseinstandarddeviationis17%.
becausethemoreelementswehavethebroaderthey
Youcanalwaystrytomodelasetof3numbers aredistributedrelativetothemean.Now,lookatDS
P.S.the17%hasbeenverifiedinExcel statements,allweneeditisjustfirststatement.So,A
issufficient.

GMATClub contributingtoeachotherslearning MATH:STATISTICS GMATClub contributingtoeachotherslearning MATH:STATISTICS

GMAT Club 2011 33


StandardDeviation5
21

SetAconsistsof19integerswithmean4and
standarddeviationof3.IfanewsetBisformedby
adding2moreelementstothesetA,whattwo
elementswilldecreasethestandarddeviationthe
most?
A)9and3

B)3and3

C)6and1

D)4and5

E)5and5

GMATClub contributingtoeachotherslearning MATH:STATISTICS

StandardDeviation5|Answers
22

Solution:Theclosertothemean,thesmaller
thestandarddeviation,andtherefore,the
greaterthedecreaseinstandarddeviation.D
has4(equaltothemean)and5(differsfrom
meanonlyby1).
Allotheroptionshaveagreaterdeviation

fromthemean
Answer:D

GMATClub contributingtoeachotherslearning MATH:STATISTICS

GMAT Club 2011 34


CompoundInterest Mixtures
1 3

If$20,000isinvestedat12%annual 14litersofapplejuiceismixedwithcranberry
interest,compoundedquarterly,whatis juice.Iftheresultingmixcontains65%of
thebalanceafter1year? cranberryjuice,howmanylitersofthemix
wereproduced?

GMATClub contributingtoeachotherslearning MATH:WORDPROBLEMS GMATClub contributingtoeachotherslearning MATH:WORDPROBLEMS

CompoundInterest|Answers Mixtures|Answers
2 4

CompoundInterestformula: Mixtureproblemsrequireattentiontodetailsto

boththeinformationgivenandthequestion.
1
Weknowthatthe14litersofapplejuiceis35%
whereC isthenumberofperiods (100%65%)ofthenewmixture.
. ConstructanX:
20,000 1 20,000 1.03
14liters35%
1.03tothe4th poweris1.1255 Xliters100%
Times20,000,thats$22,510
40 liters
(asopposedto22,400forsimpleinterest)

GMATClub contributingtoeachotherslearning MATH:WORDPROBLEMS GMATClub contributingtoeachotherslearning MATH:WORDPROBLEMS

GMAT Club 2011 35


WorkProblems WorkProblems2
5 7

Robertworkingalonecanunloadatruck
Whatistheformulaforawork in8hours.Doug,ontheotherhand,can
problem? unloadthesametruckin6hours.Ifboth
arehiredtogether,howmuchtimewillit
takeRobertandDougtounloadthe
truckworkingtogether?

GMATClub contributingtoeachotherslearning MATH:WORDPROBLEMS GMATClub contributingtoeachotherslearning MATH:WORDPROBLEMS

WorkProblems|Answers WorkProblems2|Answers
6 8

Thekeytosolvingtheworkproblems,issettingthe UsingtheWorkformula:
equationcorrectly.Theworkformulaisbasedonthe
principleofworkrates(inverseofthetimeitwould
taketocompletethejob).Theratealmostalwayswill
be
, ,

Formula:SumoftheRatesofWorkers=thecombined C= 3.4hours(approximately)
rate
;whereWistimerequiredbyworkers1
and2tocompletethejobandCisthetimerequired
forbothworkersworkingtogether(Combined)

GMATClub contributingtoeachotherslearning MATH:WORDPROBLEMS GMATClub contributingtoeachotherslearning MATH:WORDPROBLEMS

GMAT Club 2011 36


GroupProblems GroupProblems2
9 11

Outof90conferenceattendees,50registered Theofficeof120issplitbetweenmaleandfemale
forthebasicworkshopand60signedupfor employeesattheratioof3:5.If40%ofthe
theadvancedworkshop.If20attendeeshave employeesaremarriedand20ofthemarried
notsignedupforaworkshopyet,howmany employeesintheofficearemen,howmanyofthe
womenworkingintheofficearesingle?
signedupforbothadvancedandbasic
workshops?

GMATClub contributingtoeachotherslearning MATH:WORDPROBLEMS GMATClub contributingtoeachotherslearning MATH:WORDPROBLEMS

GroupProblems|Answers GroupProblems2|Answers
10 12

Thebestandeasiestapproachtosolvingthistype Toanswerthisquestionthefastest,wecanputthis
ofproblemsisusingtheboth/neitherformula tabletogether:
(alternativeoptionisaVenndiagram).
Group1+Group2+Neither Both=Total Male Female Total
Married 20 X 48
50+60+20 Both=90
Single 75 X 72
130 Both=90 Total 45 75 120
Both=40
Thekeyistomemorizetheformulaandone Thus20+X=48;X=28.Wecandotherestofthe
samplequestionsoyoucanpluginnumberson mathandfillouttheentiretabletomakesureyour
thetest calculationsarecorrect

GMATClub contributingtoeachotherslearning MATH:WORDPROBLEMS GMATClub contributingtoeachotherslearning MATH:WORDPROBLEMS


Volume/MixtureProblems(Hard) CountingProblems(UltraHard)
13 15

Ifafarmersells15ofhischickens,hisstockoffeedwill Inagameofchess,themovesofwhitesandblacks
lastfor4moredaysthanplanned,butifhebuys20 alternatewithwhiteshavingthefirstmove.During
morechickens,hewillrunoutoffeed3daysearlier
thanplanned.Ifnochickensaresoldorbought,the achesstournament,whiteshavemade2319
farmerwillbeexactlyonschedule.Howmanychickens movesaltogetherwhileblackshavemade2315
doesthefarmerhave? moves.Ifinanygamethesidethatmadethelast
12 movedidnotlose,whichofthefollowingcanbe
24 trueaboutthetournament?
48
I.Blackslost5games
55
II.Blackswonmoregamesthanwhites
60
III.Allgamesendedinadraw

GMATClub contributingtoeachotherslearning MATH:WORDPROBLEMS GMATClub contributingtoeachotherslearning MATH:WORDPROBLEMS

Volume/MixtureProblems|Answers CountingProblems|Answers
14 16

Veryhardproblem.Severalsolutionsexist;thisone Fromthestemitfollowsthattherewereonly4gamesin
isprobablynotthemostcorrectbutthequickest: whichwhiteshadthelastmove.These4gameswere
responsibleforthedifferenceinthetotalnumberofmoves
LetXbethenumberofchickensandYbethedays madebywhitesandblacksduringthetournament.We
theycansurviveonthecurrentfeed: knowthatthese4gameswerenotwonbyblacks(butthey
couldwellhaveendedinadraw).Alltheothergames
(x15)(y+4)=(x+20)(y3) couldhavebeenwonbyblacksorendedinadraw.Thus,
4 15 60 3 20 60 scenariosIIandIIIarepossible.
ScenarioIisimpossible.Itmeansthattherewereatleast5
4 15 20 3 gamesinwhichwhiteshadthelastmove.Ifthisweretrue
7 35 5 thenthedifferencebetweenthetotalnumberofmovesof
whitesandblacksshouldbeatleast5.Infact,it'sonly4.
Solvingthefirstequation,wegetx=60andy=12

GMATClub contributingtoeachotherslearning MATH:WORDPROBLEMS GMATClub contributingtoeachotherslearning MATH:WORDPROBLEMS

GMAT Club 2011 38


Distance(UltraHard) Percent
17 19

Aswimmermakesaroundtripupanddownthe Ifapricewasincreasedbyx%andthen
riverwhichtakesherXhours.Ifthenextdayshe decreasedbyy%,isthenewpricehigherthan
swimsthesamedistancewiththesamespeedin theoriginal?
stillwater,whichtakesherYhours,whichofthe
x>y
followingstatementsistrue?
x=1.2y
X>Y

X<Y

X=Y

GMATClub contributingtoeachotherslearning MATH:WORDPROBLEMS GMATClub contributingtoeachotherslearning MATH:WORDPROBLEMS

Distance|Answers Percent|Answers
18 20

Picknumbersandthencheckthemagainstthe LetPdenotetheoriginalprice.
Statement(1)byitselfisinsufficient.Ifxismuchlarger
options.Take12kmasthedistancetraveled thany,thenewpriceishigherthantheoriginal.Butifxis
up/downtheriver,andassumetheswimmer's onlymarginallylarger,thenewpriceislower.Forexample,
speedtobe4km/h;thecurrentbeing2km/h, ifx=20andy=19,thenewpriceisP*1.2*0.81=0.97P<P.
Statement(2)byitselfisinsufficient.Usethesame
whichmeans6km/hdowntheriverand2km/h reasoning.Ifyislarge,thenewpriceissmall(ify=100,the
uptheriver.Goinguprivertakes2hours,return newpriceis0).Ifyissmall,thenewpriceishigherthan
journeytakes6,thusatotalof8hours.Instill theoriginal(ifx=12andy=10,thenewpriceisP*1.12*0.9=
P*1.008>P).
water,24kmrequires6hours.ThusX=8andY=6. Statements(1)and(2)combinedareinsufficient.Adding
Plugtheseintotheanswerchoices.(8>6). S1toS2providesnonewinformation.
ThecorrectanswerisE.
ThecorrectanswerisA
GMATClub contributingtoeachotherslearning MATH:WORDPROBLEMS GMATClub contributingtoeachotherslearning MATH:WORDPROBLEMS

GMAT Club 2011 39


Rate
21

Acertainbacteriacolonydoublesinsizeeveryday
for20days,atwhichpointitreachesthelimitofits
habitatandcannolongergrow.Iftwobacteria
coloniesstartgrowingsimultaneously,howmany
dayswillittakethemtoreachthehabitatslimit?
6.33

7.5

10

15

19

GMATClub contributingtoeachotherslearning MATH:WORDPROBLEMS

Rate|Answers
22

Weknowthatthebacteriacolonydoublesinsize
everydayfor20days.Thereforeonthesecondday
itisdoublethesizeofthefirstday,andsoon.
Similarly,onthe20th day,itisat100%ofcapacity,
therefore,onthe19th day,itwillbeat50%.Since
wehave2colonies,bothwillbeoccupyinghalfof
thehabitatbythe19th day.Alternatively:
1colony= 2 ;2colonies= 2
ThecorrectanswerisE.19days.

GMATClub contributingtoeachotherslearning MATH:WORDPROBLEMS

GMAT Club 2011 40


Triangles Triangles 2
1 3

Sum of angles of ANY triangle equals ?


What can we say about sides of a triangle? List all methods for finding an area
What is the right triangle? of a triangle.

GMAT Club contributing to each others learning MATH: GEOMETRY GMAT Club contributing to each others learning MATH: GEOMETRY

Triangles | Answers Triangles 2 | Answers


2 4

Sum of all angles in any triangle is always 180


1
One side is always smaller than the sum of the 1. Area =
2
other two and is always greater than the difference
2. Heros formula: ( )( )( )
of the other two
where a,b,c are sides of a triangle and s is semi-
A right triangle is the one that has a 90 degree ::
perimeter =
angle (it has the right angle). A triangle can only 2

have one angle at 90 egrees since sum of the 3 3. If you know 2 sides of a triangle but not its height,
angles is 180 you can add an equally sized triangle to create a
square/rectangle/rhombus and find its area (may be
easier). Remember to divide your result by 2.

GMAT Club contributing to each others learning MATH: GEOMETRY GMAT Club contributing to each others learning MATH: GEOMETRY

GMAT Club 2011 41


Triangles 3 Triangles 4
5 7

These are 2 sides of a right triangle, find the third


side:
3, 4, x
What is the relationship between sides in
6, 8, x

5, 12, x
a right isosceles triangle?
12, 16, x What is the relationship between
7, 24, x angles?

GMAT Club contributing to each others learning MATH: GEOMETRY GMAT Club contributing to each others learning MATH: GEOMETRY

Triangles 3 | Answers Triangles 4 | Answers


6 8

GMAT relies on these easy triangles. If you A right isosceles triangle will have angles that are
memorize these combinations, it will save you time 90, 45, 45 degrees
on the Geometry section It will have sides that are x, x, and the hypotenuse
3, 4, 5 of 2
6, 8, 10

5, 12, 13

12, 16, 20

7, 24, 25

GMAT Club contributing to each others learning MATH: GEOMETRY GMAT Club contributing to each others learning MATH: GEOMETRY

GMAT Club 2011 42


Triangles5 Triangles6
9 11

Whatcanyouderivefromthisfigure?
(Misthecenterofacircle)
B
Nameasmanyproperties,
relationships,andformulasyou
knowaboutandequilateraltriangle A M C

GMATClub contributingtoeachotherslearning MATH:GEOMETRY GMATClub contributingtoeachotherslearning MATH:GEOMETRY

Triangles5|Answers Triangles6|Answers
10 12

Allsidesareequal ACisthediameterofacircle
Allanglesareequal whereRisradius
Area= wherea isasideofatriangle AM=MC=MB
AngleABCisarightangle
Aheightis=
Ifoneofthesidesonaninscribedtriangleisthe
Foragivenarea,anequilateraltrianglehasthe circlesdiameter,thenthisisarighttriangle
smallestperimeter itisthemostefficient regardlessofthepositionofpointB
triangle

GMATClub contributingtoeachotherslearning MATH:GEOMETRY GMATClub contributingtoeachotherslearning MATH:GEOMETRY


Triangles 7 Triangles 8 (Ultra Hard)
13 15

Is the area of the triangle ABC less than 1?


ABC < 90 degrees
What is the value of sides in a Perimeter of triangle ABC is greater than

4

30-60-90 triangle?

GMAT Club contributing to each others learning MATH: GEOMETRY GMAT Club contributing to each others learning MATH: GEOMETRY

Triangles 7 | Answers Triangles 8 | Answers


14 16
1 1 1
In 30-60-90 triangle, the sides are x, x 3, The area of the triangle ABC is 2
+ 2 =
and the hypotenuse is 2x (double the Statement (1) by itself is sufficient. In the extreme case
when Angle ABC is right, the triangle BOC is isosceles
size of the smallest side) 1
and thus 2 = and the area of the triangle ABC is a =
1. If angle ABC is smaller than 90 degrees, then the
area exceeds 1 due to the increase of the height 2 .
Statement (2) by itself is insufficient. As long as a>0,
the perimeter of the triangle ABC is always greater
4
than

The correct answer is A.

GMAT Club contributing to each others learning MATH: GEOMETRY GMAT Club contributing to each others learning MATH: GEOMETRY

GMAT Club 2011 44


Circles Circles 2
17 19

Please define the following:


Center - ?
Area of a circle = ?
Radius - ?
Length of a circle = ?
Diameter - ?

Circumference - ? = ?
Area - ?

Chord - ?

Tangent - ?

Secant - ?

GMAT Club contributing to each others learning MATH: GEOMETRY GMAT Club contributing to each others learning MATH: GEOMETRY

Circles | Answers Circles 2 | Answers


18 20

Center A point inside the circle. All points on the


circle are equidistant from the center
Radius distance between the center and any point on Area of a circle = 2
the circle. It is half the diameter
Diameter a chord passing through the center Length of a circle = 2
Circumference distance around the circle 22
Area a region enclosed by the circle = 3.14 3

7
Chord a line segment linking any two points on a
circle
Tangent line touching the circle at one point only;
tangent lines are always at 90 degrees to the radius
Secant a line that intersects a circle in 2 points
GMAT Club contributing to each others learning MATH: GEOMETRY GMAT Club contributing to each others learning MATH: GEOMETRY

GMAT Club 2011 45


Coordinate Geometry Coordinate Geometry 2
21 23

What is the equation of the If a line has a negative slope less


slope of a line? than 1 what does it say about the
line?

GMAT Club contributing to each others learning MATH: GEOMETRY GMAT Club contributing to each others learning MATH: GEOMETRY

Coordinate Geometry | Answers Coordinate Geometry 2 | Answers


22 24

Negative slope means line moves from the upper


Slope of a line equation: left hand quarant (Q2) to the bottom right hand
quadrant (Q4) or in simple terms, it is a decreasing
line. Positive slope means the opposite (duh)
2 1
= Since the slope is less than 1, it is a flat line (as
2 1 opposed to steep). Since slope is rise over run, in
this case, there is less rise than run
Where x and y are coordinates of
point 1and point 2 on that line.
GMAT Club contributing to each others learning MATH: GEOMETRY GMAT Club contributing to each others learning MATH: GEOMETRY

GMAT Club 2011 46


Coordinate Geometry 3 Coordinate Geometry 4
25 27

What slopes do Lines A, B, and C have?


Positive/Negative?

Less/Greater than 1? A
How to find the X and Y intercepts
of a line?
B

GMAT Club contributing to each others learning MATH: GEOMETRY GMAT Club contributing to each others learning MATH: GEOMETRY

Coordinate Geometry 3 | Answers Coordinate Geometry 4 | Answers


26 28

Line A Best option is to plug in the values into the


Positive Slope equation of the line
Slope greater than 1 For example, a line is y = mx + b
Line B To find the Y intercept (this is when the line crosses
Slope is neither the Y axis and thus X is zero) solve: y = b
positive or negative To find X intercept (this is when the line crosses the
Slope is Zero X axis and Y is zero) solve: 0 = mx + b
Line C The trick is to use Y = 0 when looking for X
Slope is undefined intercept and X = 0 when looking for Y intercept

GMAT Club contributing to each others learning MATH: GEOMETRY GMAT Club contributing to each others learning MATH: GEOMETRY

GMAT Club 2011 47


Coordinate Geometry 5 Coordinate Geometry 6
29 31

5
If line M with a slope of 9 goes through points A(-5, Find the equation of a line passing
-2) and B(4, 3), what is the length of the segment through the points A (5,4) and B (2,3)
AB?

GMAT Club contributing to each others learning MATH: GEOMETRY GMAT Club contributing to each others learning MATH: GEOMETRY

Coordinate Geometry 5 | Answers Coordinate Geometry 6 | Answers


30 32

The slope information in this A (5,4) and B (6,3)


irrelevant
To find an equation of a line based on two
To find distance between A ;1 ;
and B is calculated using the points, use this formula: = 1 2
;1 1 ;2
Pythagorean Theorem by
;4 4 ;3 ;4 1
drawing a triangle = ; =
;5 5 ;6 ;5 ;1
92 + 52 = 2
- + 4 = 5
81 + 25 =
= + 9
= 106

GMAT Club contributing to each others learning MATH: GEOMETRY GMAT Club contributing to each others learning MATH: GEOMETRY

GMAT Club 2011 48


Coordinate Geometry 7 Coordinate Geometry 8
33 35

If lines A and B are perpendicular to each Are the two lines below perpendicular?
other, what is the relationship between their
slopes?
A. Inverse
B. Opposite
C. Positive
D. Reciprocal
E. Reciprocal and Negative

GMAT Club contributing to each others learning MATH: GEOMETRY GMAT Club contributing to each others learning MATH: GEOMETRY

Coordinate Geometry 7 | Answers Coordinate Geometry 8 | Answers


34 36

The relationship between To answer, find the slope of each line and then
slopes of 2 perpendicular check to see if one slope is the negative reciprocal
lines is negative reciprocal of the other or if their product equals to -1.
1 5;19 ;14 14
. In other words, the Slope AB = 9;48 = ;39 = 39
two lines are perpendicular 24;4 20 20
if and only if the product of Slope CD =22;31 = ;9 = 9
their slopes is -1. 14 20
Multiply the slopes: 1;
1 39 9
E.g. 3 =-1 Not Perpendicular
3

GMAT Club contributing to each others learning MATH: GEOMETRY GMAT Club contributing to each others learning MATH: GEOMETRY

GMAT Club 2011 49


Coordinate Geometry 9 Coordinate Geometry 10 (Hard)
37 39

2 2
Does the curve + = 16
intersect the Y axis?
What is the point of intersection of two 1) 2 + 2 > 16
lines that have the following equations: 2) = +5
y=3x-3 and y=2.3x+4?

GMAT Club contributing to each others learning MATH: GEOMETRY GMAT Club contributing to each others learning MATH: GEOMETRY

Coordinate Geometry 9 | Answers Coordinate Geometry 10 | Answers


38 40

The key to solving the intersection questions is that 2 + 2 = 16 is the equation of a circle
centered at with radius 4.
at the point of intersection, both lines will have the
Statement (1) by itself is insufficient. S1 says that the
same X and Y coordinates. center of the circle is further than 4 units away from
Thus, if Y coordinates are the same, then we can the origin but it doesn't specify whether the circle is
far enough from the axis not to intersect it.
put the two equations together: 3x-3 = 2.3x+4
Statement (2) by itself is sufficient. From S2 it follows
0.7x = 7; x = 10 that and thus the center of the circle is at least 5 units
away from the axis. As the radius of the circle is only 4
Now we still need to find the Y intercept. Plug 10 units, we can conclude that the circle does not
into one of the equations: 3*10-3 = 27 intersect the axis.
Intersection point: (10, 27) The correct answer is B. Statement 2 is sufficient.

GMAT Club contributing to each others learning MATH: GEOMETRY GMAT Club contributing to each others learning MATH: GEOMETRY

GMAT Club 2011 50


CoordinateGeometry10(Hard)
45

Doesthecurve 16
intersecttheYaxis?
1 16
2 5

GMATClub contributingtoeachotherslearning MATH:GEOMETRY

CoordinateGeometry10|Answers
46

16 istheequationofacircle
centeredatwithradius4.
Statement(1)byitselfisinsufficient.S1saysthatthe
centerofthecircleisfurtherthan4unitsawayfrom
theoriginbutitdoesn'tspecifywhetherthecircleis
farenoughfromtheaxisnottointersectit.
Statement(2)byitselfissufficient.FromS2itfollows
thatandthusthecenterofthecircleisatleast5units
awayfromtheaxis.Astheradiusofthecircleisonly4
units,wecanconcludethatthecircledoesnot
intersecttheaxis.
ThecorrectanswerisB.Statement2issufficient.

GMATClub contributingtoeachotherslearning MATH:GEOMETRY

GMAT Club 2011 52


Enumeration Enumeration2
1 3

Therearethreemarbles:1blue,1grayand1
green.Inhowmanywaysisitpossibletoarrange Therearethreemarbles:1blue,1grayand1
marblesinarow?
green.Inhowmanywaysisitpossibleto
arrangemarblesinarowifblueandgreen
marbleshavetobenexttoeachother?

GMATClub contributingtoeachotherslearning MATH:PROBABILITY&COMBINATIONS GMATClub contributingtoeachotherslearning MATH:PROBABILITY&COMBINATIONS

Enumeration|Answers Enumeration2|Answers
2 4

Solution: Let'swriteoutallpossibleways Solution: Let'swriteoutallpossiblewaysto


Total:6 arrangemarblesinarawandthenfindonly
arrangementsthatsatisfyquestion'scondition:

Enumerationisamethodofcountingallpossiblewaystoarrange
elements.Althoughitisthesimplestmethod,itisoftenthefastest
methodtosolvehardGMATproblemsandisapivotalprincipleforany
othercombinatorialmethod.Infact,combinationandpermutationis Answer:4
shortcutsforenumeration.Themainideaofenumerationiswritingdown
allpossiblewaysandthencountthem.

GMATClub contributingtoeachotherslearning MATH:PROBABILITY&COMBINATIONS GMATClub contributingtoeachotherslearning MATH:PROBABILITY&COMBINATIONS

GMAT Club 2011 53


Enumeration3 Enumeration4(UltraHard)
5 7

IfN isapositiveinteger,whatisthelastdigit
of1!+2!+.N!?
Inhowmanywayscan5dressesbe Nisdivisibleby4
arrangedinastoredisplay?
isanoddinteger

GMATClub contributingtoeachotherslearning MATH:PROBABILITY&COMBINATIONS GMATClub contributingtoeachotherslearning MATH:PROBABILITY&COMBINATIONS

Enumeration3|Answers Enumeration4|Answers
6 8

1.Howmanyobjectswecanputat1stplace?5. Thisisaveryhardquestionthatrequiresanon
traditionalapproach(assomeofthehardestofficial
2.Howmanyobjectswecanputat2ndplace?4 GMATquestionsoftendo)
andsoon:3,2,1 Analyzingfactorials,youwillnoticethatthesumof
Therefore,thetotalnumberofarrangementsofn factorialswillhave3asthelastdigitifN>3,(starting
with5!,eachsumendswithazerosince5!=120,
differentobjectsinarowis 6!=720,andsoon.)
1 2 2 1 ! S1issufficientsinceweknowN>3andthuswecansay
5!=5*4*3*2*1=20*6=120 withcertaintythatlastdigitequalsto3
S2tellsusthatNisnot1or3andiseither2orgreater
N!iscalledafactorial.Factorialequalstothe than3.Ineithercase,thelastdigitwillbe3
productofnumbersfromNto1. ThecorrectanswerisD

GMATClub contributingtoeachotherslearning MATH:PROBABILITY&COMBINATIONS GMATClub contributingtoeachotherslearning MATH:PROBABILITY&COMBINATIONS

GMAT Club 2011 54


Combinations1 Combinations2
9 11

Whatisthenumberofpossiblearrangementsof
Whatisthenumberofpossiblearrangementsof objectsk inacertainorderfromacollectionof
objectsk fromacollectionofdistinctobjectsn? distinctobjectsn?

GMATClub contributingtoeachotherslearning MATH:PROBABILITY&COMBINATIONS GMATClub contributingtoeachotherslearning MATH:PROBABILITY&COMBINATIONS

Combinations1|Answers Combinations2|Answers
10 12

Acombinationisanunordered collectionofkobjects Apermutationisanordered collectionofkobjects


takenfromasetofndistinctobjects.Thenumberof takenfromasetofndistinctobjects.Thenumber
wayshowwecanchoosekobjectsoutofndistinct
objectsisdenotedas: ofwayshowwecanchoosekobjectsoutofn
Totalnumberofarrangementsofn distinctobjectsis
distinctobjectsisdenotedas:
n! 1.Thetotalnumberofarrangementsofndistinct
Nowwehavetoexcludeallarrangementsofkobjects objectsisn!
(k!) andremaining(nk) objects((nk)!) astheorderof 2.Nowwehavetoexcludeallarrangementsof
chosenk objectsandremained(nk) objectsdoesn't remaining(nk)!Objects
matter. !
!
= !
! !

GMATClub contributingtoeachotherslearning MATH:PROBABILITY&COMBINATIONS GMATClub contributingtoeachotherslearning MATH:PROBABILITY&COMBINATIONS

GMAT Club 2011 55


Combinations3 Combinations4
13 15

Whatisthedifferencebetween
combinationsandpermutations? Ifsixbusinesspartnersarehavingadinnerat
aroundtable,howmanyseating
Whentousewhichformula?
arrangementsarepossible?

GMATClub contributingtoeachotherslearning MATH:PROBABILITY&COMBINATIONS GMATClub contributingtoeachotherslearning MATH:PROBABILITY&COMBINATIONS

Combinations3|Answers Combinations4|Answers
14 16
!
Permutationsformula isusedwhen Thedifferencebetweenplacementinarowand
!
sequenceofchoicematters(meaningagroupABC thatinacircleisfollowing:ifweshiftallobjectby
isdifferentfromBACorCBA).Classicexampleis oneposition,wewillgetdifferentarrangementin
choosingnomineesfor3specificpositionsfroma arowbutthesamerelativearrangementina
poolof10candidates circle.So,forthenumberofcirculararrangements
! ofn objects,insteadofn!,wehave (n1)!
CombinationsformulaC= isusedwhen
! !
orderofselectionhasnoimpactandonceasmall Thus,theansweris5!or120
groupisformed,itdoesnotmatterhowthey
arrivedthere.Classicexampleispicking3marbles
fromabagof10
GMATClub contributingtoeachotherslearning MATH:PROBABILITY&COMBINATIONS GMATClub contributingtoeachotherslearning MATH:PROBABILITY&COMBINATIONS

GMAT Club 2011 56


Combinations5 Combinations6(UltraHard)
17 19

Inhowmanydifferentwayscanagroupof8
Ifthereare5chairsinaroomandBoband peoplebedividedinto4teamsof2people
RachelwanttositsothatBobisalwaysleftof each?
Rachel,inhowmanywaysthisseating 90

arrangementbeachieved? 105

168

420

2520

GMATClub contributingtoeachotherslearning MATH:PROBABILITY&COMBINATIONS GMATClub contributingtoeachotherslearning MATH:PROBABILITY&COMBINATIONS

Combinations5|Answers Combinations6|Answers
18 20

NotethatleftofRachel,doesnotmean Thesolutiontothisproblemisthenumberof
immediatelynexttoRachel,justleftofher. combinations.Firstwegetoneteamoutof8.The
numberofwaystodothiswouldbe .Thenext
Thisconditioniscalledsymmetrybecauseit combinationis2outof6or ,andsoon.Havingall
eliminateshalfofthepossibilities(Rachelcansit fourcombinationsmultiplied,weneedtodividethe
onlyleftorrightofBob). totalnumberbythenumberofwaystheteamscanbe
chosen,sincewearenotinterestediftheteamwith
Therefore,thenumberofwaysthatBobisleftof twocertainpeopleischosenfirst,secondorthird.
Rachelisexactly ofallpossiblewaysor Therefore,theanswerisfoundbythefollowing
formula: 105
10 !
ThecorrectanswerisB.

GMATClub contributingtoeachotherslearning MATH:PROBABILITY&COMBINATIONS GMATClub contributingtoeachotherslearning MATH:PROBABILITY&COMBINATIONS

GMAT Club 2011 57


Probability1 Probability2
21 23

WhatistheprobabilityofgettingTails when
flippingacoin?

Whatistheprobabilitythatan
eventn willoccur?
Whatistheprobabilitythatan Whatistheprobabilityofgettinga4whenrollinga
eventn willnot occur? die?

GMATClub contributingtoeachotherslearning MATH:PROBABILITY&COMBINATIONS GMATClub contributingtoeachotherslearning MATH:PROBABILITY&COMBINATIONS

Probability1|Answers Probability2|Answers
22 24

Theprobabilitythatanevenn willtakeplaceis TheprobabilityofgettingTailswhenflippingacoin


whereN isthetotalnumberofpossible is or50%sincethereare2totalpossibilitiesand
occurrences onlyoneoutcomeeachtimethecoinisflipped

Theprobabilitythatanevenn willnotoccuristhe Theprobabilityofgettinga4whencastingadieis


oppositeofitoccurring,so1 or1 p ;thereareatotalof6potentialpossibilities
(1,2,3,4,5,6)andonlyonechancetorolloneof
them.

GMATClub contributingtoeachotherslearning MATH:PROBABILITY&COMBINATIONS GMATClub contributingtoeachotherslearning MATH:PROBABILITY&COMBINATIONS

GMAT Club 2011 58


Probability3 Probability4
25 27

Ifthereisacoinandadie,whatisthe
Abucketcontains10greenand90white probabilityofgettingheadsanda"4
marbles.IfAdamrandomlychoosesamarble,
afteroneflipandonetoss?
whatistheprobabilitythatitwillbegreen?

GMATClub contributingtoeachotherslearning MATH:PROBABILITY&COMBINATIONS GMATClub contributingtoeachotherslearning MATH:PROBABILITY&COMBINATIONS

Probability3|Answers Probability4|Answers
26 28

Thenumberofgreenmarbles:n=10 Tossingacoinandrollingadieareindependent
Thenumberofallmarbles:N=10+90=100 events (occurrenceofoneeventdoesnot
Probability: 10% influenceoccurrenceofotherevents).Forn
Thereisoneimportantconceptinproblemswith independenteventstheprobabilityistheproduct
marbles/cards/balls.Whenthefirstmarbleisremoved ofallprobabilitiesofindependentevent.
fromajarandnotreplaced,theprobabilityforthe
So,theprobabilityofgettingheadsis and
secondmarblediffers( vs. ).Whereasincaseof
acoinordicetheprobabilitiesarealwaysthesame( probabilityofgettinga"4"is .Therefore,the
and ).Usually,aproblemexplicitlystates:itisa probabilityofgettingheadsanda"4"is:
problemwithreplacementorwithoutreplacement.

GMATClub contributingtoeachotherslearning MATH:PROBABILITY&COMBINATIONS GMATClub contributingtoeachotherslearning MATH:PROBABILITY&COMBINATIONS

GMAT Club 2011 59


Probability5 Probability6
29 31

Therearetwosetsofcardswithnumbers:
Ifthereisa20%chanceofrainonanaverage {1,3,6,7,8}and{3,5,2}.IfRobertchooses
day,whatistheprobabilitythatitwillrainon randomlyonecardfromthefirstsetandone
thefirstdayandwillbesunnyonthesecond? cardfromthesecondset,whatisthe
probabilityofgettingtwooddnumbers?

GMATClub contributingtoeachotherslearning MATH:PROBABILITY&COMBINATIONS GMATClub contributingtoeachotherslearning MATH:PROBABILITY&COMBINATIONS

Probability5|Answers Probability6|Answers
30 32

Theprobabilityofrainis0.2;thereforeprobability Thereisatotalof5cardsinthefirstsetand3of
ofsunshineisq=1 0.2=0.8.Thisyieldsthatthe themareodd:{1,3,7}.Therefore,theprobability
probabilityofrainonthefirstdayandsunshineon ofgettingoddcardoutofthefirstsetis .
theseconddayis:
P=0.2*0.8=0.16 Thereare3cardsinthesecondsetand2ofthem
areodd:{3,5}.Therefore,theprobabilityofgetting
Note: whenworkingwithpercents,itisimportant
toconvertthemintoadecimalformat(suchas0.2 anoddcardoutofthesecondsetis .Finally,the
for20%orafractionformatsuchas for20%) probabilityofgettingtwooddintegersis:
or40%

GMATClub contributingtoeachotherslearning MATH:PROBABILITY&COMBINATIONS GMATClub contributingtoeachotherslearning MATH:PROBABILITY&COMBINATIONS

GMAT Club 2011 60


Probability7 Probability8
33 35

Thereare8employeesincludingBoband
Rachel.If2employeesaretoberandomly
IfJessicarollsadie,whatisthe chosentoformacommittee,whatisthe
probabilityofgettingatleasta"3"? probabilitythatthecommitteeincludesboth
BobandRachel?

GMATClub contributingtoeachotherslearning MATH:PROBABILITY&COMBINATIONS GMATClub contributingtoeachotherslearning MATH:PROBABILITY&COMBINATIONS

Probability7|Answers Probability8|Answers
34 36

Twoeventsaremutuallyexclusiveiftheycannot Combinatorialapproach:
occuratthesametime.Fornmutuallyexclusive Thetotalnumberofpossiblecommitteesis =28
eventstheprobabilityisthesumofallprobabilities ThenumberofpossiblecommitteethatincludesbothBob
andRachelis1
ofevents:
P=
P(AorB)=P(A)+P(B)
Probabilityapproach:
Thereare4outcomesthatsatisfyourcondition(to
TheprobabilityofchoosingBoborRachelasafirstperson
rollatleast3):{3,4,5,6}.Theprobabilityofeach incommitteeis .TheprobabilityofchoosingRachelor
outcomeis . Bobasasecondpersonwhenfirstpersonisalreadychosen
is .TheprobabilitythatthecommitteeincludesbothBob
Theprobabilityofgettingatleasta"3"is:
andRachelis
+ +

GMATClub contributingtoeachotherslearning MATH:PROBABILITY&COMBINATIONS GMATClub contributingtoeachotherslearning MATH:PROBABILITY&COMBINATIONS

GMAT Club 2011 61


Probability8 Part2 Probability8 Part3
37 39

Thereare8employeesincludingBoband Thereare8employeesincludingBoband
Rachel.If2employeesaretoberandomly Rachel.If2employeesaretoberandomly
chosentoformacommittee,whatisthe chosentoformacommittee,whatisthe
probabilitythatthecommitteeincludesboth probabilitythatthecommitteeincludesboth
BobandRachel? BobandRachel?

GMATClub contributingtoeachotherslearning MATH:PROBABILITY&COMBINATIONS GMATClub contributingtoeachotherslearning MATH:PROBABILITY&COMBINATIONS

Probability8 Part2|Answers Probability8 Part3|Answers


38 40

ReverseCombinatorialApproach: Reverseprobabilityapproach:
Insteadofcountingprobabilityofoccurrenceofcertain
event,sometimesitisbettertocalculatetheprobabilityof Wecanchooseanyfirstperson.
theoppositeandthenuseformulap=1 q. Then,ifwehaveRachelorBobasthefirstchoice,we
Thetotalnumberofpossiblecommitteesis =28 canchooseanyotherpersonoutofthe6remaining
Thenumberofpossiblecommitteethatdoesnot includes
bothBobandRachelis:m 2 where, is
people.
thenumberofcommitteesformedfrom6remaining IfwehaveneitherRachelnorBobasfirstchoice,we
people canchooseanypersonoutoftheremaining7people.
2 isthenumberofcommitteesformedfromRobor
Rachelandoneoutof6otherpeople TheprobabilitythatthecommitteeincludesbothBob

1 1 1 1 andRachelis: 1 1 1

GMATClub contributingtoeachotherslearning MATH:PROBABILITY&COMBINATIONS GMATClub contributingtoeachotherslearning MATH:PROBABILITY&COMBINATIONS

GMAT Club 2011 62


Probability9
41

JuliaandBrainplayagameinwhichJulia
takesaballandifitisgreen,shewins.Ifthe
firstballisnotgreen,shetakesthesecond
ball(withoutreplacingfirst)andshewinsif
thetwoballsarewhiteorifthefirstballis
grayandthesecondballiswhite.Whatisthe
probabilityofJuliawinningifthejarcontains
1gray,2whiteand4greenballs?

GMATClub contributingtoeachotherslearning MATH:PROBABILITY&COMBINATIONS

Probability9|Answers
42

Sometimes,at700+levelyoumayseecomplex
probabilityproblemsthatincludeconditionsor
restrictions.Forsuchproblemsitcouldbehelpful
todrawaprobabilitytreethatincludeallpossible
outcomesandtheirprobabilities.
Now,Itisprettyobvious:

GMATClub contributingtoeachotherslearning MATH:PROBABILITY&COMBINATIONS

GMAT Club 2011 63


Recommended Math GMAT Books
Need to start from the beginning?
MGMAT Math Foundations
Need a good level of practice?
Kaplan Math Workbook
Need the most practice?
MGMAT Math Guides or Veritas Prep Math Guides
Quantitative GMAT Official Guide 2nd ed

GMAT Total Math

GMAT book reviews: http://gmatclub.com/books


GMAT Club 2011 64

GMAT Club contributing to each others learning MATH: ARITHMETIC


Best Math GMAT Courses
Kaplan Math Refresher Course - $299 or Free
Get $350 bonus when you sign up here:
http://gmatclub.com/kaplan
Kaplan Quiz Bank 1,000 math questions
Use GMAT Clubs discount to get 50% off. Visit
http://gmatclub.com/qbank for the latest promo code
Knewton full course for only $299
The best discount code plus $250 bonus
http://gmatclub.com/knewton

GMAT Club 2011 65

GMAT Club contributing to each others learning MATH: ARITHMETIC


VERBAL STRATEGIES

GMAT Club 2011 66


ContentsofFlashCards
2

BasicStrategiesandPrinciplesofSentence
Correction,CriticalReasoning,andReading
Comprehensionwithafewexamples

Illustrationoferrorsandrightanswerchoices
throughexamples

GMAT Club 2011 67


GMATClub contributingtoeachotherslearning VERBAL
BasicDeconstruction
3

Step1:Readthequestionstemandcategorizethequestion.

Step2:Readthestimulusandidentifythepremiseandthe
CriticalReasoning conclusion

Step3:Trytofocusontheconclusionandthinkofanswerchoices
thatmightberight

Step4:Useprocessofeliminationtoruleoutwronganswer
choices.Don'ttrytomakethemfit!

Step5:Makesureanswerchoicemakessense!

GMATClub contributingtoeachotherslearning VERBAL

Contents MainpartsofaCRquestion
2 4

MainPoint/MustBeTrue
Weaken Conclusion
Strengthen
Assumption
ResolvetheParadox
Reasoning
Premise Assumption
BoldFaced
NumbersandPercents

GMATClub contributingtoeachotherslearning VERBAL GMATClub contributingtoeachotherslearning VERBAL

GMAT Club 2011 68


MainpartsofaCRquestion TypeI AscertainConclusion
5 7

Conclusion:Thisisthefinalargumentthatthe Thesearequestionswhereweassumethatthe
authormakes. stimulusistrue andtrytofindanswerchoicesthat
aresupportedbytheconclusion
Premise:Theseareevidentiarystatementsthat
supporttheconclusion PossibleQuestionTypes:
1. Inference
Assumption:Theseareunstatedpremises,on 2. MainPoint
whichtheconclusionandsometimesthepremise 3. MustbeTrue
reston.

GMATClub contributingtoeachotherslearning VERBAL GMATClub contributingtoeachotherslearning VERBAL

Premise&Conclusion TypeII Strengthen&Support


6 8

PREMISE CONCLUSION Thesearequestionswhereweassumethatthe


givenanswerchoicesaretrue andtrytopickthe
Supportstheconclusion Answers Hasatoneoffinality andconveys bestonethatwillsupportthestimulus
thequestionofWhy? thefinalmessageofwhatthe
authorissaying
Because Thus PossibleQuestionTypes:
Since Therefore 1. Strengthen
For/Forthereason Hence 2. Assumption
Dueto So
Asindicatedby Asaresultof/Consequently
Furthermore Accordingly
Giventhat Itfollowsthat/Itmustbethat
GMATClub contributingtoeachotherslearning VERBAL GMATClub contributingtoeachotherslearning VERBAL
TypeIII WeakenandHurt MostCommonMistakeTypes
9 11

OutofScope/IrrelevantAnswers
Thisisbasicallytheoppositeoftheabovetypeand
Talksaboutsomethingcompletelyirrelevanttothe
aimstodisprovetheconclusionofthestimulus.
discussionathand.
Hencewetaketheanswerchoicetobetrue here
Peopletendtopickthesewhentheyrecompletely
aswell. unsureofwhattheyresupposedtobedoing

PossibleQuestionTypes: ToneMismatchAnswers
1. Strengthen Answersthatdontagreewiththetoneofthepassage

2. Assumption Mightbetoostrongortooweakwhencomparedto
stimulus

GMATClub contributingtoeachotherslearning VERBAL GMATClub contributingtoeachotherslearning VERBAL

MostCommonMistakeTypes MainPoint/MustBeTrue
10 12

OppositeAnswers
Whichofthefollowingrepresentsthemainideaoftheparagraph?
Doestheoppositeofwhattheanswerchoiceis Whichofthefollowingcanbeinferredfromtheabove?
supposedtodo
Peoplepickthembecausetheymightgetconfused CorrectAnswerChoices
aboutthequestiontype
Canthisanswerchoicebeprovenorvalidatedbywhatisgivenin
thestimulus?IsthisanswerchoicetruetothestimulusANDthe
ShellGameAnswers mainpointofthepassage,i.e.similartotheconclusion?
Shouldbevalidatedbythestimulus(Stimulusistakentobetrue)
Soundsreallysimilartothestimulusbutdiffersbyjust
Shouldbethemainpointofthestimulus,notjustapremise(for
therightamounttobeincorrectbutstilllucrative. MainPointquestions)
Peoplepickthesewhentheyrenotpayingclose Willeitherrestateconclusionorpresentitinadifferentmanner
attention
GMATClub contributingtoeachotherslearning VERBAL GMATClub contributingtoeachotherslearning VERBAL

GMAT Club 2011 70


ABCDE Weaken
13 15

Tobeconsideredforthisyearsmeritscholarshipaward,studentsneedto
haveperfectattendanceanda4.0GPA.Alexistheonlypersonintheclass Whichofthefollowing,iftrue,callsintoquestionthevalidityofthe
whohasa4.0buthehashad5absences. argument?
Whichofthefollowingmostseriouslyunderminestheconclusion?
Theclaimsabove,iftrue,moststronglysupportwhichofthefollowing
conclusions? CorrectAnswerChoices

Doesthisanswerchoicebreakdowncausality?Doesitgivean
A. Nostudentatthisschoolhasperfectattendancefortheyear alternatecause,showthatthecauseeffectrelationshipisnon
B. Somestudentsatthisschoolwhodidnothavea4.0alsodidnothave existentorreversed?
perfectattendance Answerchoiceshouldbreakdownstructureofstimulus(Answeris
C. Alexistheonlystudentwhocouldbeconsideredfortheaward takentobetrue)
D. Nostudentatthisschoolqualifiesfortheawardthisyear Couldbeinrelationtoagrossgeneralization,awrongconclusion
orincorrecthypothesisfromfacts.
E. Manystudentshaveachievedperfectattendancebutnever4.0GPAs.

GMATClub contributingtoeachotherslearning VERBAL GMATClub contributingtoeachotherslearning VERBAL

ABCDE
14 16

A:Exaggeration.
B:Possible,butnotnecessary
C:Thestimulusclearlysaysthatyouneedboth
perfectattendanceand4.0GPA.
D:Thisistrue.IfAlexistheonlyonewhohasa
4.0andhedoesnthaveaperfectattendance,it
meanstheentireschoolisbarredfrom
qualifyingfortheaward.Correctchoice.
E: Outofscopeanswer.

GMATClub contributingtoeachotherslearning VERBAL GMATClub contributingtoeachotherslearning VERBAL

GMAT Club 2011 71


ABCDE Strengthen
17 19

Thereare350brandsofcellphonesinthemarkettoday.However,ourstore
onlystocksthetop10brands.Inordertoincreaseoursales,weplanto Whichofthefollowing,iftrue,strengthenstheargumentthemost?
increasethesizeofourinventorytothetop50brands. Whichofthefollowing,iftrue,wouldmostsignificantlyhelpprovethe
scientistshypothesis?
Whichofthefollowing,iftrue,pointsoutamajorflawintheplanabove?
CorrectAnswerChoices
A. Thecapabilitiesofthetopfivecellphonesarealmostthesame,withno
brandhavingconsistentsuperiorityinallrespects. Doesthisanswerchoicereinforcetheconclusiondirectly?Doesit
B. Thetop8brandsaccountforalmostallthecellphonessold validateanassumptionorruleoutadiscrepancy?Doesithelpestablish
causality?
C. Asusersgetmoresophisticated,theywanttotryoutthelesserknown Answerchoiceshouldstrengthenstructureofstimulus(Answeristaken
brandswhichmightoffersomeothervaluetothem. tobetrue)
D. Lesspopularbrandsprovidelittleprofittothestorebecausetheyhaveto Needstodirectlystrengthenconclusionbybridginglogicalgaps,
bediscountedtobesold validatingreasonsorassumptionsorfindingmissinglinks.Ifitsnota
directstrengthening,moveon!Donttrytomaketheconnections!
E. Theleadingbrandsarenowlosingsalestolesspopularbrandsthatoffer
similarfeaturesforalowercost
GMATClub contributingtoeachotherslearning VERBAL GMATClub contributingtoeachotherslearning VERBAL

ABCDE
18 20

A:Irrelevant.Doesthisaffectprofitmarginsforthe
storeiftheyweretoincreaseinventory?No
B:Thismeansthatthestorealreadyhasthebrands
thatsellthemost.Increasinginventorywillhave
littleeffectonprofitmargins.CorrectAnswer.
C:Thisalmoststrengthenstheargument.
D:Thistellsyouthatthelesserbrandswillprovide
lesserprofit.Butlessisarelativeterm.Thisisan
unclearstatement.
E: Thisalmoststrengthenstheargumentbyfavoring
anincreaseininventory.Henceincorrect.
GMATClub contributingtoeachotherslearning VERBAL GMATClub contributingtoeachotherslearning VERBAL

GMAT Club 2011 72


ABCDE Assumption
21 23

Recently,severalcompanieshavewithdrawntheiradsfromMagazineA,becausethe
editorialboardofthemagazinehaddecidedtochangetheimagethatthemagazine Theauthorassumeswhichofthefollowinginsayingthat
portraysfromoneoffamilyvaluestooneconcernedmorewithsexandviolence.Surely
thisindicatesthatthedecisionmakersinadvertisingagenciesdostillhaveasenseofmoral Theargumentcannotbetrueunlesswhichofthefollowing
proprietythatoccasionallydrivestheiractions. statementsareassumed?

Whichofthefollowing,iftrue,wouldstrengthenthisconclusion? CorrectAnswerChoices
A)Theadvertisersregularlyreviewtheplacementoftheiradvertisements.
B)Itisarareeventforseveraladvertiserstowithdrawalltheiradvertisements Supporter:Linksunrelatedelementsinthestimulusandfillsin
simultaneouslyfromapublication. logicalgaps
C)Theadvertisers,whenquestioned,admittedthattheirclientswouldloserevenueasa Defender:Eliminatesthealternativesandanychoicesthatmight
resultoftheadvertisementsbeingwithdrawn. weakentheconclusion.
D)Theadvertisersallplacednewadvertisementswithotherpublicationsthatemphasised
familyvalues. AssumptionNegationTechnique:Narrowitdowntothefinal
E)AsurveyofthereadershipofMagazineXsuggestedthatthemajorityofthereadership answerchoicesandthennegatethem themainmodifierora
thinkthatthestandardofthemagazine'scontentshadfailedsinceitstransformation. verb(suchthatthemeaningofthesentenceisreversed)andifthe
negatedchoiceweakenstheconclusion,theanswerisright.
QuestionfromGMATClub(95810)
GMATClub
contributingtoeachotherslearning VERBAL GMATClub contributingtoeachotherslearning VERBAL

ABCDE
22 24

A:Thisisirrelevanttothequestionofmoral
propriety.
B:Thisdoesntnecessarilypointtomoral
proprietydirectly.Dontmakeunnecessary
connections!
C:Onceagain,nocorrelationtowhatweretalking
about
D:Ifthisistrue,thenitshowsthattheagencies
careaboutwheretheiradsgo.Hencecorrect.
E: Thisisaboutthemagazine.Nottheadvertisers.

GMATClub contributingtoeachotherslearning VERBAL GMATClub contributingtoeachotherslearning VERBAL

GMAT Club 2011 73


ABCDE ResolvetheParadox
25 27

Intheory,itspossiblethatbacteriadevelopedonMarsearlyinitshistoryandsome
werecarriedtoEarthbyameteorite.However,strainsofbacteriafromdifferent
planetswouldprobablyhavesubstantialdifferencesinproteinstructurethatwould
Whichofthefollowing,iftrue,helpsexplaintheparadoxabove?
persistovertime,andnotwobacterialstrainsonEartharedifferentenoughtohave Whichofthefollowing,iftrue,helpsexplaintheapparent
arisenondifferentplanets.So,evenifbacteriadidarriveonEarthfromMars,they discrepancyintheargument?
musthavediedout.
CorrectAnswerChoices
Theargumentismostvulnerabletowhichofthefollowingcriticisms?
ActiveResolution:Donttrytodisprovethestimulus,takeitasa
A.ItfailstoestablishwhetherbacteriaactuallydevelopedonMars. given.
B.ItfailstoestablishhowlikelyitisthatMartianbacteriaweretransportedtoEarth.
C.Itfailstoconsiderwhetherthereweremeansotherthanmeteoritesbywhich
Doestheanswerchoiceaddressthefacts?Theanswerchoice
MUSTconformtothestimulus
MartianbacteriacouldhavebeencarriedtoEarth.
D.ItfailstoconsiderwhetherallbacterianowonEarthcouldhavearisenfrom TheanswershouldaddressBOTHsidesoftheparadox,and
transportedMartianbacteria. resolveit.Itshouldntstrengthentheparadox.
E.Itfailstoconsiderwhethertherecouldhavebeenstrainsofbacteriathat
originatedonEarthandlaterdiedout.
GMATClub contributingtoeachotherslearning
QuestionfromGMATClub(80726)
VERBAL GMATClub contributingtoeachotherslearning VERBAL

ABCDE
26 28

A:Thisisirrelevanttotheargumentthatstatesthat
evenifbacteriacamefromMars,theymusthave
diedout.
B:OutofScope!
C:Again,thisdoesnttalkaboutbacteriastrainsdying
out
D:Ifthisistrue,thenthereneednothavetobeany
significantdifferencebetweenthestrainsfoundon
Earth,sincetheyalloriginatedfromMars.
E: Wearenotconcernedaboutthis.

GMATClub contributingtoeachotherslearning VERBAL GMATClub contributingtoeachotherslearning VERBAL

GMAT Club 2011 74


ABCDE Reasoning
29 31

AseveredroughtcanlessenthetotalamountofgovernmentaidthatUSfarmersreceive
asagroup.Thegovernmentpaysfarmerstheamount,ifany,bywhichthemarketpriceat
whichcropsareactuallysoldfallsshortofapresettargetpriceperbushelforthecrops. Whichofthefollowingmostaccuratelydescribestheauthors
Thedroughtof1983,forexample,causedfarmprogrampaymentstodropby$10billion. methodofdefendingthecase?
Whichofthefollowingismostparalleltotheargumentinlogical
Giventheinformationabove,whichofthefollowing,iftrue,bestexplainswhythe structure?
droughtof1983resultedinareductioninfarmprogrampayments?
A. Priortothedroughtof1983,thegovernmentraisedthetargetpriceforcropsinorderto CorrectAnswerChoices
aidfarmersinreducingtheirdebtloads.
B. Duetothedroughtof1983,UnitedStatesfarmersexportedlessfoodin1983thaninthe
precedingyear. Shouldfollowthesamepatternofreasoning.Focusonthelogic,
C. Duetothedroughtof1983,UnitedStatesfarmershadsmallerharvestsandthusreceived
notthecontentoftheargument!
ahighermarketpriceforthe1983cropthanforthelargercropoftheprecedingyear. Answerchoicemustbeadescriptionoftheauthorslogic!
D. Duetothedroughtof1983,UnitedStatesfarmersplannedtoplantsmallercropsin1984 Answerchoiceshouldnotbeinconsistentwiththestimulus,even
thantheyhadin1983. ifthestimulushasflawedlogic.Wearenotaskedtoevaluatethe
E. Despitethedroughtof1983,retailpricesforfooddidnotincreasesignificantlybetween logicofthestimulus;wearemerelyaskedtofindananswerthat
1982and1983.QuestionfromGMATClub(80726) followsthesamepattern rightornot.
GMATClub contributingtoeachotherslearning VERBAL GMATClub contributingtoeachotherslearning VERBAL

ABCDE
30 32

A:Strengthenstheparadox.
AmountPaid=Target Market.Wecantsay
anythingunlessweknowwhathappenedtothe
marketprice.
B:Exportsareirrelevanthere.
C:Thistalksaboutthehighermarketprices.Thiscould
explaintheparadoxbecausetarget marketwill
becomelesserifmarketbecomeshigher
D:Sizeofcropsisirrelevant.
E: Between19821983?Retailprice?OutofScope!

GMATClub contributingtoeachotherslearning VERBAL GMATClub contributingtoeachotherslearning VERBAL

GMAT Club 2011 75


ABCDE BoldFacedQuestions
33 35
Shakespearewastheworld'sgreatestplaywright,whichmeansthathemusthavewrittentheworld's
greatestplays.Thefactthathewrotethegreatestplaysintheworldonlygoestoprovethatheisthe
world'sgreatestplaywright. Thetwoboldfacedsentencesplaywhichofthefollowing
Whichofthefollowingstatementsbestmatchestheargumentshownabove? roles?
A. Dr.SmithistheonlymemberofthetwentystrongEnglishdepartmentwhothinksthatByronwrote
greaterpoetrythanKeats.ThereforeByronwasnotasgoodapoetasKeatswas. CorrectAnswerChoices
B. Amiscarriageofjusticeoccurswhenaninnocentpersonissenttoprison.However,thereareno
miscarriagesofjusticebecausepeopleinprisonareguilty.Otherwisetheywouldnothavebeensent
toprison.
Shouldaddressboththeboldfacedportions,notjustone.
C. Thepresenceofenlargedwhitecorpusclesinthebloodwouldindicatethatthepatientmustbe
sufferingfromdiseaseX.However,hisbloodsamplecontainsnoenlargedwhitecorpuscles,sohe Willbetruetothestimulusandmimicthereasoningin
cannotbesufferingfromdiseaseX. thestimulus!
D. Blairdenieshavingcommittedtherobbery,buthasimplicatedBrown.Brownontheotherhand,
claimsthathewaselsewhereatthetime,andpointsthefingersquarelybackatBlair. Answerchoiceshouldnotbeinconsistentwiththe
E. Ibelievethatthepresenceofthetartarsaucehelpstoaccentuatetheflavouroftheother stimulus,anditshouldfollowthesamelogicalpatternin
ingredients,buttheyinturnreleasechemicalswhichtendtoneutralisethetartarsauce.Therefore ordertoarriveattheargumentofthestimulus..
thedishismuchlessspicythanpuretartarsaucewouldbe.
QuestionfromGMATClub(92697)
GMATClub contributingtoeachotherslearning VERBAL GMATClub contributingtoeachotherslearning VERBAL

ABCDE
34 36

A:Thiscallsformakingajudgmentbasedon
someonesopinion.Nottrue.
B: Thisistrue.Theargumentinthestimulusisa
circularargumentthatsaysAprovesBandthen
usesBtoproveA.Asimilarpatternexistshere.
C:Thishasanegationeffectinthereasoning.Not
relevant.
D:Shellgameanswer.Mightseemlikecircularlogic,
butitsactuallyjustacircularsequenceofevents.
E: Somekindofcontradictoryreasoning.Notrelevant.

GMATClub contributingtoeachotherslearning VERBAL GMATClub contributingtoeachotherslearning VERBAL

GMAT Club 2011 76


ABCDE Numbers&Percents
37 39
AlthoughtheearliestsurvivingGreekinscriptionswritteninanalphabetdatefromtheeighthcentury
B.C.,astrongcasecanbemadethattheGreeksactuallyadoptedalphabeticwritingatleasttwo Larger numbers doesnt mean larger percentage
centuriesearlier. Significantly,thetextoftheseearliestsurvivingGreekinscriptionssometimesruns
fromrighttoleftandsometimesfromlefttoright.Now,theGreekslearnedalphabeticwritingfrom and smaller number doesnt mean smaller
thePhoenicians,andintheprocesstheywouldsurelyhaveadoptedwhateverconventionthe
Phoenicianswerethenusingwithrespecttothedirectionofwriting. Originally,Phoenicianwriting
percentage. For example,
ranineitherdirection,butbytheeighthcenturyB.C.ithadbeenconsistentlywrittenfromrightto
leftforabouttwocenturies.

Intheargumentgiven,thetwoportionsinboldfaceplaywhichofthefollowingroles? Increase in percentage is not the same as saying


A.Thefirstisthepositionthattheargumentseekstoestablish;thesecondreportsadiscoverythat there is an increase in absolute numbers. This
hasbeenusedtosupportapositionthattheargumentopposes.
B.Thefirstisthepositionthattheargumentseekstoestablish;thesecondpresentsanassumption
could merely be the result of a change in the
onwhichtheargumentrelies.
C.Thefirstpresentsevidencethatisusedinsupportofthepositionthattheargumentseeksto
group size. For example, 5% of 100 > 10% of 50.
establish;thesecondpresentsanassumptiononwhichtheargumentrelies.
D.Thefirstisanobjectionraisedagainstapositionthattheargumentopposes;thesecondisthe
positionthattheargumentseekstoestablish.
E.Thefirstisanobjectionraisedagainstapositionthattheargumentopposes;thesecondis
evidencethathasbeenusedtosupportthatposition. QuestionfromGMATClub(98749)
GMATClub contributingtoeachotherslearning VERBAL GMATClub contributingtoeachotherslearning VERBAL

ABCDE
38 40

A:Thesecondboldfacedportiondoesntrepresenta
discovery;itsanassumption
B: Thisistrue.Thefirstpartisaclausethestimulus
wantstoproveandthesecondisanassumption
C:Thefirstboldfacedportiondoesntobjecttoany
statement.
D:Thefirstboldfacedportiondoesntobjecttoany
statement.
E: Thefirstboldfacedportiondoesntobjecttoany
statement.

GMATClub contributingtoeachotherslearning VERBAL GMATClub contributingtoeachotherslearning VERBAL

GMAT Club 2011 77


ABCDE
41

ArecentsurveyofallautoaccidentvictimsinDoleCountyfoundthat,oftheseverelyinjured
driversandfrontseatpassengers,80percentwerenotwearingseatbeltsatthetimeoftheir
accidents.Thisindicatesthat,bywearingseatbelts,driversandfrontseatpassengerscangreatly
reducetheirriskofbeingseverelyinjurediftheyareinanautoaccident.

Theconclusionaboveisnotproperlydrawnunlesswhichofthefollowingistrue? ReadingComprehension
A. Ofallthedriversandfrontseatpassengersinthesurvey,morethan20percentwerewearingseat
beltsatthetimeoftheiraccidents.
B. Considerablymorethan20percentofdriversandfrontseatpassengersinDoleCountyalways
wearseatbeltswhentravellingbycar.
C. Moredriversandfrontseatpassengersinthesurveythanrearseatpassengerswereveryseverely
injured.
D. Morethanhalfofthedriversandfrontseatpassengersinthesurveywerenotwearingseatbelts
atthetimeoftheiraccidents.
E. MostoftheautoaccidentsreportedtopoliceinDoleCountydonotinvolveanyseriousinjury.

QuestionfromGMATClub(88036)
GMATClub contributingtoeachotherslearning VERBAL

ABCDE Contents
42 44

A:Thisisatrickyquestion.Ouraimistoproveacorrelation.Letssay100
peoplewereseverelyinjuredand100werenot.Outofthe100 GlobalQuestions
severelyinjured,80didntwearseatbelts.
Probabilityofapersonnotwearingseatbelttogetinjured= MainPoint/PrimaryPurpose

PassageOrganization
Probabilityofapersonwearingseatbelttogetinjured=
AuthorsPerspective/PassageTone

B:Doesntestablishthecorrelationbetweenwhatsbeingsaid.Hence LocalQuestions
incorrect.
C:Thisdoesntevengiveusarelativeindication.Incorrect. SpecificReference
D:Thistellsusaboutthenumberofpeoplewhowerenotwearingseat Function
belts,butnotaboutthenumberofpeoplewhowereinjured.
E: Completelyirrelevanttothescopeofthediscussion. StrengthenandWeaken

ParallelReasoningQuestion

GMATClub contributingtoeachotherslearning VERBAL GMATClub contributingtoeachotherslearning VERBAL


FourQuestionsforRC CommonIndicators
45 47

Readfromgeneraltospecificatthreelevels.Changeyourreading

strategy,notyourreadingspeed.Answerthefollowingquestions. Main PointorStrongPurpose
Why?MainPointofthepassage.
VeryCommonQuestionType
How?Structureofthepassage Introduction,Exampleand
CounterExample.Andsoon.
Primarygoalofreadingpassage Findthemain
What?Whatisbeingsaid?(MainPointofIndividualParagraphs) point!
WhatTone?Makesureanswerchoicemakessense!

GMATClub contributingtoeachotherslearning VERBAL GMATClub contributingtoeachotherslearning VERBAL

Newvs.ExistingIdeas CommonIndicators
46 48

CONTINUATIONOFOLDIDEAS INTRODUCTION OFNEWIDEAS


DifficultWords,Phrases&Concepts
Continues elaboratingonanideathats Introducesanothernewidea, perhaps
alreadybeenpresented tocontrastsomethingpresented. VeryCommonDistraction
Furthermore However

ForInstance But Dontfocusonthedifficultyofthewordsor


ForExample Nevertheless terminologies
Additionally Yet

Similarly InContrast
Instead,choosetofocusontheunderlying
Indeed Although

Infact Still
meaningofwhatitssaying
GMATClub contributingtoeachotherslearning VERBAL GMATClub contributingtoeachotherslearning VERBAL

GMAT Club 2011 79


CommonIndicators CommonIndicators
49 51

ListofThings/Enumerations DatesandNumbers
Paycloseattention! Matchthecorrectdateswiththeevents
mentioned
Dontmemorize!
Perhaps,makeanoteofthedatesandashort
Verycommonquestionindicator.Makeamental handversionoftheeventonyournotepad.
noteofwherethelistoccurs,soyoucanreturnto
it,ifnecessary.

GMATClub contributingtoeachotherslearning VERBAL GMATClub contributingtoeachotherslearning VERBAL

CommonIndicators CommonIndicators
50 52

Reference withAuthority HiddenReferences


Ifauthoritiesarementioned,thinkabouthowandwhy Ideasthatarementionedmorethanonceinthe
thisauthoritativeremarkisnecessary. passage.

Mightrepresentconflictingviewpointsorideas.
Evenifthequestioncitesalinenumberforoneof
thereferences,theanswerchoicemightdrawon
Makenoteofwhateachauthoritysaysandwhytheir
authorityisrelevant. thesameideamentionedelsewhere,oranother
partofthepassage.
Very,veryimportant,andveryeasytomisinterpret!
GMATClub contributingtoeachotherslearning VERBAL GMATClub contributingtoeachotherslearning VERBAL

GMAT Club 2011 80


CommonIndicators BasicQuestionTypes
53 55

Contrasting Views
Ifseveralviewpointsarepresentedinthepassage,
makenoteofeachpointandwhossayingit/why Type
itsbeingsaid.

Understandingofthesecounterexamplesorviews
areveryimportant!Theywillbeindicatedby Global Local
wordssuchasHoweverorIncontrast

GMATClub contributingtoeachotherslearning VERBAL GMATClub contributingtoeachotherslearning VERBAL

CommonIndicators QuestionTypes
54 56

Definitions
Global BroadQuestions
Commonwhenthepassageisofscientificnature

Makeanoteofthedefinitionandexpecttobe MainPoint/PrimaryPurpose
questionedaboutyourunderstandingofthe
definition PassageOrganization

AuthorsPerspective/PassageTone

GMATClub contributingtoeachotherslearning VERBAL GMATClub contributingtoeachotherslearning VERBAL

GMAT Club 2011 81


BroadQuestions BroadQuestions
57 59

MainPoint/Primary Purpose AuthorsPerspective/ToneofPassage


Thesequestionsaskyoutoreflectonanauthors
RepresentsCoreIdeas perspective

Willaskaboutthebroadermeaningofthe Understandwhattheauthoristryingtosay,and
passage,andwhatitseekstoconvey. whereheorshestandswithrespecttotheviews
presented.

Istheauthoraggressive?Oristheauthordocile?
Whatisthetoneofthemessageconveyed?
GMATClub contributingtoeachotherslearning VERBAL GMATClub contributingtoeachotherslearning VERBAL

BroadQuestions QuestionTypes
58 60

Passage Organization Local SpecificQuestions(FourTypes)


Thiswillaskaboutthestructureofapassage SpecificReference

Forinstance,thestructuremightbesomething Function
likethis:
Introduction
Example StrengthenandWeaken
CounterExample

ParallelReasoningQuestion
GMATClub contributingtoeachotherslearning VERBAL GMATClub contributingtoeachotherslearning VERBAL

GMAT Club 2011 82


LocalQuestions LocalQuestions
61 63

SpecificReference Strengthen/Weaken
Willrefertoaspecificlineorparagraphinthe SimilartoCriticalReasoningQuestionsofthe
passageandaskforaquestionrelatingtothat. sametype.

Mightinvolvecrossreferencingwithother Therequiredanswerwilleitherhelporhurtthe
relevantinformationpresentedelsewhereinthe conclusiveviewpointpresentedbytheauthor,
passage i.e.themainpoint

Assumetheanswerchoicesgiventobetrue.
GMATClub contributingtoeachotherslearning VERBAL GMATClub contributingtoeachotherslearning VERBAL

LocalQuestions LocalQuestions
62 64

Function Parallel Reasoning


Questionsaboutwhatapieceofthepassage Again,similartoCriticalReasoningQuestionofthe
sametype
eitheraparagraph,alineorevenawordistrying
toaccomplishwithrespecttothebroaderscope Willasktoidentifyanaction,amonggivenactions,or
ofthepassage aviewpointthatmimicsthereasoningthatthe
authorfollows.

Trytoinfersomethingaboutthecontextandtone Identifyingthestructureofthepassageandthe
ofthespecificreference,andaskyourself structureofalinewithinapassageasnecessary
Whatdoesthislinedo? wouldgreatlyhelp!
GMATClub contributingtoeachotherslearning VERBAL GMATClub contributingtoeachotherslearning VERBAL

GMAT Club 2011 83


RCinaNutshell
65

Therightmentality:ThepassageWILLbe
intentionallyconfusing.Getusedtoit!
SentenceCorrection
Awarenessofcontent:Thepassagesmightbe
fromhumanities,socialsciencesorsciences.
Dontgetboggeddownbyonekindorget
excitedaboutanother

ReadingPattern:GeneraltoSpecific
GMATClub contributingtoeachotherslearning VERBAL

RCinaNutshell Contents
66 68

Understandquestiontypes:GlobalorLocal SubjectVerbAgreement
VerbTenseErrors
NounAgreement
Prephrase:Framearoughanswerbeforeyou
pickanswerchoices! Pronouns
Modifiers
Comparisons
ProcessofElimination:Dontkeepananswer
Parallelism
thatyouhavetomakeconnectionsfor.

GMATClub contributingtoeachotherslearning VERBAL GMATClub contributingtoeachotherslearning VERBAL

GMAT Club 2011 84


DeconstructingSC Andwhatsnot?
69 71

Step1:Readthequestionstemandthinkofpossibleerrorsinthe
sentence,subjectverbagreement,tensemismatchetc.
ThreeQuestionTypesYouWontSee
Step2:Readtheanswerchoiceandsplititintotwogroupsbasedon
Spelling TheGMATwillnottestyouonyour
overallstructure. knowledgeofspellings

Step3:Oneofthegroupswillcontainanerror.Eliminatethegroup
andresplitthenextgroup. Punctuation Addingacommaoranapostrophe
andsimilarthingswillnotbetested.Semicolons,
Step4:Useprocessofeliminationtoruleoutwronganswerchoices. however,aretested.
Don'ttrytomakethemfit!

Step5:Makesuretheanswerchoicemakessense! Capitalization TheGMATdoesnttestyouon


yourknowledgeofcapitalizationeither.
GMATClub contributingtoeachotherslearning VERBAL GMATClub contributingtoeachotherslearning VERBAL

WhatsTested? ErrorsTested
70 72

ThreeQuestionTypesYouWillSee SubjectVerbAgreement
Grammar Thesentencehastoadheretotherules Thisdealswiththeissueofplurality.
ofgrammarfollowedbyStandardEnglish.

Singularsubjectsmustusesingularverbs.For
Meaning Thesentencehastohavearelevant
meaningandconveyitproperly example:Hewas eating.

Concision Ifwhatisbeingsaidcanbesaidinfive PluralVerbsmustusepluralverbs.Forexample:


wordsinsteadoften,thentheformerispreferred Theelephantswere walking.
(Activevs.Passivevoice,forexample)
GMATClub contributingtoeachotherslearning VERBAL GMATClub contributingtoeachotherslearning VERBAL

GMAT Club 2011 85


Trap1: Phrasesbetweensubject andverb Trap3: MultipleNounsorPronouns
73 75

Removetheadditionalinformationandreadthe Ifthereismorethanonenounortheusageofa
sentencewithoutthem. pronouninasentence,thenthesubjectverb
BarelyseventeenandleadingtheFrencharmywearing agreementMUSTbeconsistent!!
amansarmor,JoanofArc,anilliteratepeasantgirl
fromtheFrenchcountryside,brokethesevenmonth TwoExceptions:
oldseizeofOrleansinninedays. Conjunctions(OR,NOR)AlwaysSINGULAR
Readingthesentencewithoutthatpartwehave: UsageofEACHorEVERY AlwaysSINGULAR
BarelyseventeenandleadingtheFrencharmywearing
amansarmor,JoanofArc,brokethesevenmonthold
seizeofOrleansinninedays.
GMATClub contributingtoeachotherslearning VERBAL GMATClub contributingtoeachotherslearning VERBAL

Trap2: SubjectFollows Verb Trap4: Indefinite Pronouns


74 76

Ifthereareexpletives,thencheckforsubjectverb Pronounslikeall,any,more,most,somebody,nobody
agreement,byrearrangingthesentence. andsoon.
Somecommonexpletives: Pluralityisbasedonwhattheindefinitepronounis
There referringto!(Thisistheantecedent)
Here Exceptions:
It Each,Either,Neither,Every,Number Alwayssingular
Or irrespectiveofwhatfollows!
Example:AnumberofreasonsPlural!Thenumber of
cars Singular!("the" is singular and "a" is plural)

GMATClub contributingtoeachotherslearning VERBAL GMATClub contributingtoeachotherslearning VERBAL

GMAT Club 2011 86


ErrorsTested ErrorsTested
77 79

VerbTense Noun Agreement


Tense Forms Thenumberofnounsmustbeconsistentwithwhat
theyarereferencing.
Past Simple
Incorrect: MattandDavebelievedthattheirhard
workintheirengineeringclasswillhelpthemrealize
Present Perfect theirdreamofbecomingagreatengineer.

Future Progressive Correct:MattandDavebelievedthattheirhardwork


intheirengineeringclasswillhelpthemrealizetheir
dreamofbecominggreatengineers.
PerfectProgressive
GMATClub contributingtoeachotherslearning VERBAL GMATClub contributingtoeachotherslearning VERBAL

ErrorsTested ErrorsTested
78 80

VerbTense SubTypes UseofPronouns


IncorrectVerbTense
PronounAntecedentDisagreement

ShiftinVerbTense
IncorrectUseofRelativePronouns

VerbVoice
AmbiguousandImpliedPronouns

GMATClub contributingtoeachotherslearning VERBAL GMATClub contributingtoeachotherslearning VERBAL

GMAT Club 2011 87


PronounErrorSubTypes Trap 2: Indefinite Pronouns
81 83

PronounAntecedentDisagreement
Theseareverygeneralpronouns many,few,
ThepronounMUSTrefertoitsantecedent. both,everyandsoon.

Thepronounantecedentrelationshipshouldbe Incorrect:Manyofthestudentsweresurprisedto
consistentthroughoutthesentence. learnthathisorherexamwasunfairlygraded.
Incorrect: Eachofthewomenselectedforthe
scholarshipwereaskedtosubmitanapplication. Correct:Manyofthestudentswe
surprisedtolearnthattheirexamswasunfairly
Correct: Eachofthewomenselectedforthe graded.
scholarshipwasaskedtosubmitanapplication.
GMATClub contributingtoeachotherslearning VERBAL GMATClub contributingtoeachotherslearning VERBAL

Trap 1: Distancebtw.pronoun&antecedent Trap 3: Misleading Antecedents


82 84

Thishappenswhenthereisadditionalinformation
betweenthepronounandantecedentmakingiteasy AntecedentsthatSOUNDSpluralbutactualare
tolosetrackoftherelationshipbetweenpronouns
andtheirantecedents.
singularorviceversa.

Incorrect:Thelibrary,withitsmanybooksand Forexample,familyissingular,andpersonsis
databases,requireaspecialmembership. singulartoo.Itmightbeagroupofpeopleina
family,butbeingacollectivenoun,itsstill
Correct: Thelibrary,withitsmanybooksand singular!
databases,requiresaspecialmembership.

GMATClub contributingtoeachotherslearning VERBAL GMATClub contributingtoeachotherslearning VERBAL

GMAT Club 2011 88


PronounErrorSubTypes Trap 2: Whovs.Whom?
85 87

Incorrectuseofrelativepronouns
Verysimpletrick!
Theyrelategroupsofwordstoanounorpronoun Makethefollowingsubstitutions:
which,whom,whomsoever,whereandwhy. He Who
Him Whom

Twotrapsofincorrectusage: Thesubstitutionthatmakessensewillcorrespondwith
IncorrectPronounChoiceforPeopleandThings thecorrectwordinthatcontext.
Differencebetweenwhoandwhom
Sometimesitmightbenecessarytorearrangethe
sentenceforittomakesense.
GMATClub contributingtoeachotherslearning VERBAL GMATClub contributingtoeachotherslearning VERBAL

Trap 1: PronounChoice ErrorsTested


86 88

Modifiers
WhoandWhomwillalwaysrelatetopeople.
Misplacementofadjectives(describesnouns
Thedoctorthat performedthesurgerywashonored serene)andadverbs(describeverbs serenely)
shouldbewho

Forexample:ThenewBMWmodelisdesignedto
ThatandWhichwillalwaysrelatetothings,i.e. drivefasterandmoreefficientthantheprevious
inanimateobjects. model.
Themachinewho performedthesurgerywas Theusageofthewordefficientiswrongheresinceits
showcasedintheexhibit shouldbethat modifyingtheverb.Thecorrectwordusagewouldbe
efficiently
GMATClub contributingtoeachotherslearning VERBAL GMATClub contributingtoeachotherslearning VERBAL

GMAT Club 2011 89


Trap1: Quantifiers Trap2: Placement
89 91

ErrorsinCountNouns(canbecounted trees, Misplacedmodifierswithrelativeclauses(like


bottles,billions).Quantifiersaremany,both,few, thatorwhich)
several,acoupleandsoon.
Incorrect:Accordingtothereport,bicycleswillbe
ErrorsinNonCountNouns(cannotbecounted confiscatedthathavenotbeenregisteredwiththe
water,liquid).Quantifiersaremuch,alittle,abit university.
andsoon.
Correct:Accordingtothereport,bicyclesthathave
Somemodifierslikeall,andsomecanbeusedfor notbeenregisteredwiththeuniversitywillbe
both. confiscated.

GMATClub contributingtoeachotherslearning VERBAL GMATClub contributingtoeachotherslearning VERBAL

Trap2: Placement ErrorsTested


90 92

Misplacedmodifiersaremodifiersthatareplacedin

thewrongpositiononasentence. Comparisons
Incorrect:Shortonmoney,thecarwasthebestoneTanya
couldfind. Incorrectuseofcomparativedegree
Correct:Shortonmoney,Tanyaknewthatthecarwasthe
bestoneshecouldfind.
IllogicalComparisons
DanglingModifiersarethosewherethereferentis
completelyabsent.
Incorrect: Walkingtotheuniversity,thecathadtostop.
Correct:Walkingtotheuniversity,hehadtostopbecause
ofhiscat.
GMATClub contributingtoeachotherslearning VERBAL GMATClub contributingtoeachotherslearning VERBAL

GMAT Club 2011 90


ComparisonSubType ErrorsTested
93 95

Degree ofComparison Parallelism


Whentherearetwoobjectsbeingcompared,youhave ParallelVerbsandVerbFormErrors
touseacomparativeword.Whenmorethantwo
objectsarecompared,youhavetouseasuperlative ParallelNounErrors
word.
Incorrect:Thoughthereportersfeltthatinjuryplayedapart ParallelPrepositionsandArticles
inTennesseeslosstoMinnesota,Tennesseescoachsaid
thatMinnesotahadthebest teamthatnight. ParallelConjunctions
Correct:Thoughthereportersfeltthatinjuryplayedapart
inTennesseeslosstoMinnesota,Tennesseescoachsaid ParallelComparisons
thatMinnesotahadthebetterteamthatnight.
GMATClub contributingtoeachotherslearning VERBAL GMATClub contributingtoeachotherslearning VERBAL

ComparisonSubType ParallelismSubType
94 96

Illogical Comparison VerbErrors


Onlysimilarobjectscanbecompared.Youcannot Nounsarenaturallyparallel.Butwhentheyarein
compareahumantoadog. theformwithaningattheend(gerunds),theyare
Incorrect: ThoughtherecentLaysfoodpoisoning confusedforverbs.
incidenthascausedsomebuyerstoquestionitsquality,
theLayschipsaresoldmoreoftenthananyotherfood Incorrect: Thecommonsymptomsofthestomachflu
company. areindigestion,vomitinganddrinkinglesswaterthan
Correct: ThoughtherecentLaysfoodpoisoningincident
usual.
hascausedsomebuyerstoquestionitsquality,theLays
chipsaresoldmoreoftenthanthoseproducedbyany Correct: Thecommonsymptomsofthestomachflue
otherfoodcompany. areindigestion,vomitinganddecreasedwaterintake.
GMATClub contributingtoeachotherslearning VERBAL GMATClub contributingtoeachotherslearning VERBAL

GMAT Club 2011 91


ParallelismSubType ParallelismSubType
97 99

NounErrors ParallelConjunctions
Whenasentencehastwoormoresimilarparts Theconjunctionsusedmustbeparallelinstructure.
linkedbyaconjunction,eachparthastobeofthe Someexamples:
sameverbform. Neither nor Eitheror
Incorrect:Thenewexampatternrequiresstudentsto Notonly butalso Bothand
entertheirstudentIDsandsigningtheirnamesonthe Whether or Asas
roster.
Correct: Thenewexampatternrequiresstudentsto Incorrect: Therestaurantnotonlyhiredanewchef,butit
entertheirstudentIDsandsigntheirnamesonthe also expandeditsseatingcapacity.
roster. Correct: Therestaurantnotonlyhiredanewchef,butalso
expandeditsseatingcapacity.
GMATClub contributingtoeachotherslearning VERBAL GMATClub contributingtoeachotherslearning VERBAL

ParallelismSubType ParallelismSubType
98 100

Prepositional/Article Errors ParallelComparisons


Aprepositionandanarticlemusteitherbeusedby
allpartsofasentenceorbyjustthefirstpart. Whencomparisons areleftunparallel,thatcan
amount toanerroraswell.
Incorrect: Bydoingeachassignment,turningin Incorrect: In thebook,theauthormentionsthathe
homeworksontimeandby attendingalltheclasses, foundeditingpicturesmuchmorearduousand
onemightbeabletoobtainthehighestgradeinthe difficultthanwhenhehadtotaketheactualpictures.
class. Correct: Inthebook,theauthormentionsthathe
foundeditingpictures muchmorearduousand
Correct: Bydoingeachassignment,turningin difficultthantakingpictures.
homeworksontime,andattendingalltheclasses,one
mightbeabletoobtainthehighestgradeintheclass.
GMATClub contributingtoeachotherslearning VERBAL GMATClub contributingtoeachotherslearning VERBAL

GMAT Club 2011 92


ErrorsTested ErrorsTested
101 103

SemiColons Idioms
Usageoffragmentsordependentclauseson
eithersideofasemicolon Very,verycommonerrortested.
Aphrasethatiscommonlyacceptedascorrect
Incorrect:Theguineapigisnotatruepig;rather,a
eventhoughitsgrammaticallyinconsistent
rodentthatbelongstotheCaviidae family. Anidiomerrorwillmisrepresenttheidiom.
Correct: Theguineapigisnotatruepig,butrathera Idiomsareprovidedwithrightandwrongusage
rodentthatbelongstotheCaviidae family. inthecardsthatfollow.
Correct: Theguineapigisnotatruepig;itsarodent
thatbelongstotheCaviidae family.
GMATClub contributingtoeachotherslearning VERBAL GMATClub contributingtoeachotherslearning VERBAL

ErrorsTested ErrorsTestedIdiomRules
102 104

WordinessandRedundancy Amongvs.Between
TheGMATwillpreferusageofthemostconcise
sentencespossible.
Among:
Incorrect: Hedecidedtonotpurchasethecarduetothe Used when more than two items are in question.
factthat itwastooexpensive.
Example: He was the best among three candidates.
Correct: Hedecidedtonotpurchasethecarbecauseit
wastooexpensive.

Incorrect: Inadditiontoplayingtheguitar,Robalso plays Between:


theviola. Used when two items are in question
Correct: Inadditiontoplayingtheguitar,Robplaysthe
viola. Example: He was the best between the two of them.
GMATClub contributingtoeachotherslearning VERBAL GMATClub contributingtoeachotherslearning VERBAL

GMAT Club 2011 93


ErrorsTestedIdiomRules ErrorsTestedIdiomRules
105 107

Bothvs.Each EachOthervs.OneAnother

Both: Each Other:


Used to point out similarities.
Used to compare two things.
Example: Both of them were good at swimming.
Example: They loved each other dearly.

Each:
Used to point out differences/dissimilarities. (Always
One another:
singular) Used to compare more than two things.

Example: Each girl had her own niche. Example: The three brothers loved one another dearly.
GMATClub contributingtoeachotherslearning VERBAL GMATClub contributingtoeachotherslearning VERBAL

ErrorsTestedIdiomRules ErrorsTestedIdiomRules
106 108

Twicevs.Double Ifvs.Whether

Twice: If:
Twice/Thrice etc. are used for comparison Primarily used only in If else sentences.
Example: The Toyota was twice as fast as the Honda.
Example: If this is true, the market will collapse. Else, it will
be okay tomorrow.

Double:
Whether:
Used as a verb only.
Used more frequently as a comparison.
Example: He more than doubled his wealth by
Example: Whether or not he chooses to accept the prize is
investing in stocks. up to him.
GMATClub contributingtoeachotherslearning VERBAL GMATClub contributingtoeachotherslearning VERBAL

GMAT Club 2011 94


ErrorsTestedIdiomRules ErrorsTestedIdioms
109 111

Agreeupon/to
Likevs.SuchAs
Correct: Weagreeduponthedateofthemeeting

Incorrect:Weagreedtothedateofthemeeting.
Like:
Used when indicating similarities between things. Correct:Weagreedtoshareourroom.
Example: Like John, Amy was a violinist too. Incorrect:Weagreeduponsharingourroom.

Allowfor
Such as:
Correct: Onemustalwaysallowforamarginoferror
Used to list examples. whilecalculatingaxialloads.
Example: There are different kinds of tigers such as the Incorrect:Onemustalwaysallowtohaveamarginof
Bengal Tigers, white tigers and so on. errorwhilecalculatingaxialloads.
GMATClub contributingtoeachotherslearning VERBAL GMATClub contributingtoeachotherslearning VERBAL

ErrorsTestedIdioms ErrorsTestedIdioms
110 112

AbilityTo Appealto
Correct:Dolphinshavetheabilitytoemitlowfrequency Correct:Iappealedtohissenseofjustice.
whistles
Incorrect:Iappealedforhissenseofjustice.
Incorrect:Dolphinshavetheabilityofemittinglow
frequencywhistles.
Areindangerof
Actas/like
Correct:Thegiantpandabearsareindangerof
Correct:Asignaturecanactasa legalattetato.
extinction.
Incorrect:Asignaturecanactlikealegalattetato.
Incorrect:Thegiantpandabearshaveadangerof

Correct:Hewasactinglikeachild. dyingfromdeforestation.
Incorrect:Hewasactingasachild.
GMATClub contributingtoeachotherslearning VERBAL GMATClub contributingtoeachotherslearning VERBAL

GMAT Club 2011 95


ErrorsTestedIdioms ErrorsTestedIdioms
113 115

Asanadolescent Attributeto
Correct: Asanadolescent,hesufferedfromattention Correct:Iattributemysuccesstohardwork.

disorder Incorrect:Iattributemysuccesswithhardwork.

Incorrect:Whileinadolescence,hesufferedfrom

attentiondisorder. Baseon
Correct:Thedecisionwasmadebasedonmultiple
Asgoodas criteria.
Correct:Itsasgoodasnew. Incorrect:Thedecisionwasmadebasedofmultiple
criteria.

GMATClub contributingtoeachotherslearning VERBAL GMATClub contributingtoeachotherslearning VERBAL

ErrorsTestedIdioms ErrorsTestedIdioms
114 116

Associatewith Begintoseedaylight
Correct:Iassociatespringwithflowers. Correct:Afterworkingontheprojectallnight,Iam
finallybeginningtoseedaylight.
Incorrect:IalwaysassociatewatertoNiagarafalls.
Incorrect:Afterworkingontheprojectallnight,Iam
finallybeginningtoviewthedaylight.
Attendto
Correct:Ihavetoattendtosomeduties Between<>and<>
Incorrect:Ihavetoattendforsomeduties. Correct:Ihadtochoosebetweenchocolatesand
cakes.
Incorrect:Ihadtochoosebetweenchocolateswith
cakes.
GMATClub contributingtoeachotherslearning VERBAL GMATClub contributingtoeachotherslearning VERBAL

GMAT Club 2011 96


ErrorsTestedIdioms ErrorsTestedIdioms
117 119

Carefor/about Consider(as)
Correct:Idontcaremuchforsweets
Correct:Idontconsiderhimaseriouscontender
Correct:Icareaboutheralot.
Incorrect:Idontconsiderhimwithaserious

contender
Claimto/Claimthat
Correct:Ineverclaimedtopossessremarkablesinging
talent. Conformto
Incorrect:Ineverclaimedthatpossessremarkable Correct:Youmustconformtothestandards
singingtalent.
Incorrect:Youmustconformwiththestandards
Correct:IneverclaimedthatIpossessedremarkable
talent.
GMATClub contributingtoeachotherslearning VERBAL GMATClub contributingtoeachotherslearning VERBAL

ErrorsTestedIdioms ErrorsTestedIdioms
118 120

Cometoadeadend Contrastto/with
Correct:Hecametoadeadendafterresearchingtopics. Correct:Incontrasttothepreviousreport,thisoneisbetter

Incorrect: Hecamebyadeadendafterresearchingtopics. Incorrect:Incontrastwiththepreviousreport,thisoneis


better.
Compareto/with
Correct:Inevercomparedmyselftoher. Correct:Theredbackgroundcontrastsnicelywiththeblue
flowers
Incorrect:Inevercomparedmyselfforher.
Incorrect:Theredbackgroundcontrastsnicelytotheblue
flowers.
Correct:Letscomparetheprosofthesituationwiththe
cons.
Counton
Incorrect:Letscomparetheprosofthesituationforthe
cons. Correct:IknewthatIcouldcountonyou.

Incorrect:IknewthatIcouldcountforyou.
GMATClub contributingtoeachotherslearning VERBAL GMATClub contributingtoeachotherslearning VERBAL

GMAT Club 2011 97


ErrorsTestedIdioms ErrorsTestedIdioms
121 123

Creditfor/to/with Declared<>
Correct:Youshouldtakecreditforwhatyoudid. Correct:Theteacherdeclaredallexamsworthy.

Incorrect:Youshouldtakecredittowhatyoudid. Incorrect:Theteacherdeclaredallexamsasworthy.

Correct:Icreditmysuccesstomyhardwork. Defineas
Incorrect:Icreditmysuccessformyhardwork.
Correct:Lightisdefinedasradiation.

Incorrect:Lightisdefinedforradiation.
Correct:Heiscreditedwiththediscoveryofpenicillin.
Incorrect:Heiscreditedfor/tothediscoveryofpenicillin.

GMATClub contributingtoeachotherslearning VERBAL GMATClub contributingtoeachotherslearning VERBAL

ErrorsTestedIdioms ErrorsTestedIdioms
122 124

Debateabout Delightedto
Correct:Therewasaragingdebateaboutstandards. Correct:Iamdelightedtoacceptthisprize.

Incorrect:Therewasaragingdebateforstandards. Incorrect:Iamdelightedforacceptingthisprize.

Decideon Differentfrom
Correct:Iamyettodecideonacolorforthewall. Correct:Thissandwichisdifferentfromtheothers.

Incorrect:Iamyettodecideaboutacolorforthe Incorrect:Thissandwichisdifferentwiththeothers.
wall.

GMATClub contributingtoeachotherslearning VERBAL GMATClub contributingtoeachotherslearning VERBAL

GMAT Club 2011 98


ErrorsTestedIdioms ErrorsTestedIdioms
125 127

Distinguishbetween/from Drawto
Correct:Ihadtodistinguishbetweengoodandbad. Correct:Hewasdrawntoherfromthemomenthe
Incorrect:Ihadtodistinguishfromgoodandbad. mether.
Incorrect:Hewasdrawnforherfromthemomenthe
Correct:Ihadtodistinguishgoodfrombad. mether.
Incorrect:Ihadtodistinguishgoodwithbad.
Easiersaidthandone
Drawaline Correct:Itsalwayseasiersaidthandone.
Correct:Wehadtodrawalinesomewhere.
Incorrect:Itsalwayseasiersaidthantobedone.
Incorrect:Wehadtodrawthelinebefore.
GMATClub contributingtoeachotherslearning VERBAL GMATClub contributingtoeachotherslearning VERBAL

ErrorsTestedIdioms ErrorsTestedIdioms
126 128

Drawattentionto Electas/to
Correct:Ihatetodrawattentiontothefactthatthe Correct:Hewaselectedtooffice.

qualityoffoodhasgonedown. Incorrect:Hewaselectedtoofficer.

Incorrect:Ihatetodrawattentionforthefactthatthe
qualityoffoodhasgonedown. Correct:Hewaselectedasanofficer.
Incorrect:Hewaselectedasoffice.
Drawupon
Correct:Wehadtodrawuponthereservestokeepthe
carrunning. Easiersaidthandone
Correct:Itsalwayseasiersaidthandone.
Incorrect:Wehadtodrawonthereservestokeepthe
carrunning. Incorrect:Itsalwayseasiersaidthantobedone.

GMATClub contributingtoeachotherslearning VERBAL GMATClub contributingtoeachotherslearning VERBAL

GMAT Club 2011 99


ErrorsTestedIdioms ErrorsTestedIdioms
129 131

Indicatethat Left,rightandcenter
Correct:Studiesindicatethatstressisacommoncause Correct:Shewasshootingemailstopeopleleft,rightand
ofheartattacks. center.
Incorrect:Studiesindicateaboutstressbeingacommon
Incorrect:Shewas,leftrightandcenter,sendingemails.
causeofheartattacks.

Inorderto Knownto
Correct:Shebeganstudyinginordertogetagoodgrade Correct:Evenasastudent,Rickwasknowntodothings

intheclass. differently.
Incorrect:Shebeganstudyinginorderthatshegota Incorrect:Evenasastudent,Rickwasknownaswantingto
goodgradeintheclass. dothingsdifferently.
GMATClub contributingtoeachotherslearning VERBAL GMATClub contributingtoeachotherslearning VERBAL

ErrorsTestedIdioms ErrorsTestedIdioms
130 132

Justas<>,so<> Ameansto
Correct:JustasTyrawasconsideredforthescholarship,so Correct:Sheonlyvieweditasameanstotheend.
wasMia. Incorrect:Forsomepeople,moneyisconsidereda
Incorrect:JustasTyrawasconsideredforthescholarship,
meansfor/ofanend.
Miaasalsoconsidered.

Mistakenfor
Knownto
Correct:Thetwinswereoftenmistakenforone
Correct:Evenasastudent,Rickwasknowntodothings
differently. another.
Incorrect:Thetwinswereoftenmistakenasone
Incorrect:Evenasastudent,Rickwasknownaswantingto
dothingsdifferently. another.
GMATClub contributingtoeachotherslearning VERBAL GMATClub contributingtoeachotherslearning VERBAL

GMAT Club 2011 100


ErrorsTestedIdioms ErrorsTestedIdioms
133 135

Morethanever Reluctantto
Correct:IregretnotgoingtoEurope,nowmorethanever.
Correct:Shewasreluctanttotakeonsuchahugetask.
Incorrect:IregretnotgoingtoEurope,nowmorethan
Incorrect:Shewasreluctantabouttakingonsucha
never.
hugetask.
Nativeof/to
Correct:SheisanativeofVienna. Requireof
Incorrect:SheisanativetoVienna. Correct:Itisrequiredofallcandidatestoreporthere
daily.
Correct:TheBengalTigerisnativetoIndia. Incorrect:Itisrequiredfromallcandidatestoreport
Incorrect:TheBengalTigerisnativeofIndia. heredaily.
GMATClub contributingtoeachotherslearning VERBAL GMATClub contributingtoeachotherslearning VERBAL

ErrorsTestedIdioms ErrorsTestedIdioms
134 136

Prohibitfrom Seemto
Correct:Onlywhenweprohibitpeoplefrombuyingdrinks
Correct:Heseemedtobehidingsomethinginhispocket.
forminors,willwereducetheproblemofunderage
drinking. Incorrect:Heseemedashewashidingsomethinginhis

Incorrect:Onlywhenweprohibitpeopletobuydrinksfor pocket.
minors,willwereducetheproblemofunderagedrinking.
Takeadvantageof
Rangefrom
Correct:Thequalityoftheseproductsrangefromgoodto Correct:Hetookadvantageofalltheopportunitieshehad.

excellent. Incorrect:Hetookadvantageforalltheopportunitieshe
Incorrect:Thequalityoftheseproductsrangebetween had.
goodtoexcellent.
GMATClub contributingtoeachotherslearning VERBAL GMATClub contributingtoeachotherslearning VERBAL

GMAT Club 2011 101


Idioms Idioms
137 139

Whichsentenceisidiomaticallycorrect? Whichsentenceisidiomaticallycorrect?
A. WillFerrell'scameoscenesaresofunnyas A. Themarkethasconfidencethat thefirmwilladd
anythinghe'severdone. back leveragetoincreaseprofits.
B. WillFerrell'scameoscenesareasfunnythat B. Themarkethasconfidencein thefirm'sabilityto
anythinghe'severdone. addback leveragetoincreaseprofits.
C. WillFerrell'scameoscenesareasfunnyas C. Themarkethasconfidencein thefirmtoadd
anythinghe'severdone. back leveragetoincreaseprofits.

GMATClub contributingtoeachotherslearning VERBAL GMATClub contributingtoeachotherslearning VERBAL

Idioms Idioms
138 140

Correctanswer: Correctanswer:
A. WillFerrell'scameoscenesaresofunnyas A. Themarkethasconfidencethat thefirmwilladd
anythinghe'severdone. back leveragetoincreaseprofits.
B. WillFerrell'scameoscenesareasfunnythat B. Themarkethasconfidencein thefirm'sabilityto
anythinghe'severdone. addback leveragetoincreaseprofits.
C. WillFerrell'scameoscenesareasfunnyas C. Themarkethasconfidencein thefirmtoadd
anythinghe'severdone. back leveragetoincreaseprofits.

GMATClub contributingtoeachotherslearning VERBAL GMATClub contributingtoeachotherslearning VERBAL

GMAT Club 2011 102


Idioms Idioms
141 143

Whichsentenceisidiomaticallycorrect? Whichsentenceisidiomaticallycorrect?
A. Whatthesescorestellusisthatrigorislackingin A. Thecityindustrialistshavedemanded thestate
someschools. governmenttomeet thepromisesmadebychief
ministerAshokChavan.
B. Whatthesescorestellusisthatsomeschools
B. Thecityindustrialistshavedemandedthat the
lackof rigor. stategovernmentmeet thepromisesmadeby
C. Whatthesescorestellusisthatthere'salackof chiefministerAshokChavan.
rigorinsomeschools. C. Thecityindustrialistshavedemandedthat the
stategovernmentmet thepromisesmadeby
chiefministerAshokChavan.

GMATClub contributingtoeachotherslearning VERBAL GMATClub contributingtoeachotherslearning VERBAL

Idioms Idioms
142 144

Correctanswer: Correctanswer:
A. Whatthesescorestellusisthatrigorislackingin A. Thecityindustrialistshavedemanded thestate
someschools. governmenttomeet thepromisesmadebychief
ministerAshokChavan.
B. Whatthesescorestellusisthatsomeschools
B. Thecityindustrialistshavedemandedthat the
lackof rigor. stategovernmentmeet thepromisesmadeby
C. Whatthesescorestellusisthatthere'salackof chiefministerAshokChavan.
rigorinsomeschools. C. Thecityindustrialistshavedemandedthat the
stategovernmentmet thepromisesmadeby
chiefministerAshokChavan.

GMATClub contributingtoeachotherslearning VERBAL GMATClub contributingtoeachotherslearning VERBAL

GMAT Club 2011 103


Idioms Idioms
145 147

Whichsentenceisidiomaticallycorrect? Whichsentenceisidiomaticallycorrect?
A. Tensionshaveflaredinsomepartsofthecountry A. TheproblemisDisney'sabilitytotake controlof
between blackswith Hispanics. someofthebestknowncharactersquicklyisvery
limited.
B. Tensionshaveflaredinsomepartsofthecountry
B. TheproblemisDisney'sabilityoftaking control
between blacksand Hispanics. ofsomeofthebestknowncharactersquicklyis
C. Tensionshaveflaredinsomepartsofthecountry verylimited.
among blacksand Hispanics. C. TheproblemisDisney'sabilityfortaking control
ofsomeofthebestknowncharactersquicklyis
verylimited.

GMATClub contributingtoeachotherslearning VERBAL GMATClub contributingtoeachotherslearning VERBAL

Idioms Idioms
146 148

Correctanswer: Correctanswer:
A. Tensionshaveflaredinsomepartsofthecountry A. TheproblemisDisney'sabilitytotake controlof
between blackswith Hispanics. someofthebestknowncharactersquicklyisvery
limited.
B. Tensionshaveflaredinsomepartsofthecountry
B. TheproblemisDisney'sabilityoftaking control
between blacksand Hispanics. ofsomeofthebestknowncharactersquicklyis
C. Tensionshaveflaredinsomepartsofthecountry verylimited.
among blacksand Hispanics. C. TheproblemisDisney'sabilityfortaking control
ofsomeofthebestknowncharactersquicklyis
verylimited.

GMATClub contributingtoeachotherslearning VERBAL GMATClub contributingtoeachotherslearning VERBAL

GMAT Club 2011 104


Idioms Idioms
149 151

Whichsentenceisidiomaticallycorrect? Whichsentenceisidiomaticallycorrect?
A. Areadingabove50percentindicates the A. Incontrastto thefirsttwooptionscentralizing

manufacturingeconomytobe generally decisionmaking,aCapandTradesystemwillnot.


expanding. B. Ascontrastedwith thefirsttwooptions,aCap
B. Areadingabove50percentindicatesthat the andTradesystemwilldecentralize decision
manufacturingeconomyis generallyexpanding. making.
C. Areadingabove50percentindicates the C. Incontrastto thefirsttwooptions,aCapand
manufacturingeconomyis generallyexpanding. Tradesystemwilldecentralize decisionmaking.

GMATClub contributingtoeachotherslearning VERBAL GMATClub contributingtoeachotherslearning VERBAL

Idioms Idioms
150 152

Correctanswer: Correctanswer:
A. Areadingabove50percentindicates the A. Incontrastto thefirsttwooptionscentralizing
manufacturingeconomytobe generally decisionmaking,aCapandTradesystemwillnot.
expanding. B. Ascontrastedwith thefirsttwooptions,aCap
B. Areadingabove50percentindicatesthat the andTradesystemwilldecentralize decision
manufacturingeconomyis generallyexpanding. making.
C. Areadingabove50percentindicates the C. Incontrastto thefirsttwooptions,aCapand
manufacturingeconomyis generallyexpanding. Tradesystemwilldecentralize decisionmaking.

GMATClub contributingtoeachotherslearning VERBAL GMATClub contributingtoeachotherslearning VERBAL

GMAT Club 2011 105


Idioms Idioms
153 155

Whichsentenceisidiomaticallycorrect? Whichsentenceisidiomaticallycorrect?
A. Thisreporthasbeenissuedbytheassociation A. Itsgrossmarginrosefrom 22%upto 25%,butits
since1931,excepting afouryearinterruption operatingmarginfellfrom 7%downto 4%.
duringWorldWarII.
B. Itsgrossmarginrosefrom 22%to 25%,butits
B. Thisreporthasbeenissuedbytheassociation
since1931,exceptfor afouryearinterruption operatingmarginfellfrom 7%to 4%.
duringWorldWarII. C. Itsgrossmarginrosefrom 22%until 25%,butits
C. Thisreporthasbeenissuedbytheassociation operatingmarginfellfrom 7%till 4%.
since1931,withtheexceptionof afouryear
interruptionduringWorldWarII.

GMATClub contributingtoeachotherslearning VERBAL GMATClub contributingtoeachotherslearning VERBAL

Idioms Idioms
154 156

Correctanswer: Correctanswer:
A. Thisreporthasbeenissuedbytheassociation A. Itsgrossmarginrosefrom 22%upto 25%,butits
since1931,excepting afouryearinterruption operatingmarginfellfrom 7%downto 4%.
duringWorldWarII.
B. Itsgrossmarginrosefrom 22%to 25%,butits
B. Thisreporthasbeenissuedbytheassociation
since1931,exceptfor afouryearinterruption operatingmarginfellfrom 7%to 4%.
duringWorldWarII. C. Itsgrossmarginrosefrom 22%until 25%,butits
C. Thisreporthasbeenissuedbytheassociation operatingmarginfellfrom 7%till 4%.
since1931,withtheexceptionof afouryear
interruptionduringWorldWarII.

GMATClub contributingtoeachotherslearning VERBAL GMATClub contributingtoeachotherslearning VERBAL

GMAT Club 2011 106


Idioms Idioms
157 159

Whichsentenceisidiomaticallycorrect? Whichsentenceisidiomaticallycorrect?
A. "Ibelievethatwealthisameansfor anend,not A. TheancientChineseTaoistsdistinguished
anendinitself". intercourseand orgasm.
B. "Ibelievethatwealthisameansof anend,not B. TheancientChineseTaoistsdistinguished
anendinitself". intercoursefrom orgasm.
C. "Ibelievethatwealthisameansto anend,not C. TheancientChineseTaoistsdistinguished
anendinitself". between intercourseand orgasm.

GMATClub contributingtoeachotherslearning VERBAL GMATClub contributingtoeachotherslearning VERBAL

Idioms Idioms
158 160

Correctanswer: Correctanswer:
A. "Ibelievethatwealthisameansfor anend,not A. TheancientChineseTaoistsdistinguished
anendinitself". intercourseand orgasm.
B. "Ibelievethatwealthisameansof anend,not B. TheancientChineseTaoistsdistinguished
anendinitself". intercoursefrom orgasm.
C. "Ibelievethatwealthisameansto anend,not C. TheancientChineseTaoistsdistinguished
anendinitself". between intercourseand orgasm.

GMATClub contributingtoeachotherslearning VERBAL GMATClub contributingtoeachotherslearning VERBAL

GMAT Club 2011 107


Idioms Idioms
161 163

Whichsentenceisidiomaticallycorrect? Whichsentenceisidiomaticallycorrect?
A. Itwouldhavebeeneasyenoughfor hertobuy a A. iPhoneusersdownloadtwotofourtimesas
ranchstylehousethatshecouldeasilyandlogically manymore games,video,andotherWebdata
decoratewithScandinavianstylefurniture. than othersmartphoneusers.
B. Itwouldhavebeeneasyenoughsothat shebought B. iPhoneusersdownloadtwotofourtimesas
aranchstylehousethatshecouldeasilyandlogically manymore games,video,andotherWebdataas
decoratewithScandinavianstylefurniture. othersmartphoneusers.
C. Itwouldhavebeeneasyenoughastobuy aranch C. iPhoneusersdownloadtwotofourtimesas
stylehousethatshecouldeasilyandlogically many games,video,andotherWebdataas other
decoratewithScandinavianstylefurniture. smartphoneusers.

GMATClub contributingtoeachotherslearning VERBAL GMATClub contributingtoeachotherslearning VERBAL

Idioms Idioms
162 164

Correctanswer: Correctanswer:
A. Itwouldhavebeeneasyenoughfor hertobuy a A. iPhoneusersdownloadtwotofourtimesas
ranchstylehousethatshecouldeasilyandlogically manymore games,video,andotherWebdata
decoratewithScandinavianstylefurniture. than othersmartphoneusers.
B. Itwouldhavebeeneasyenoughsothat shebought B. iPhoneusersdownloadtwotofourtimesas
aranchstylehousethatshecouldeasilyandlogically manymore games,video,andotherWebdataas
decoratewithScandinavianstylefurniture. othersmartphoneusers.
C. Itwouldhavebeeneasyenoughastobuy aranch C. iPhoneusersdownloadtwotofourtimesas
stylehousethatshecouldeasilyandlogically many games,video,andotherWebdataas other
decoratewithScandinavianstylefurniture. smartphoneusers.

GMATClub contributingtoeachotherslearning VERBAL GMATClub contributingtoeachotherslearning VERBAL

GMAT Club 2011 108


Idioms Idioms
165 167

Whichsentenceisidiomaticallycorrect? Whichsentenceisidiomaticallycorrect?
A. Wehaveonechanceinamillionof winningthe A. Oureconomicwellbeingisgenerallydetermined
WorldCup. from theamountofgoodsandservicesthatwe
consume.
B. Wehaveoneinamillionchancesto winthe
B. Oureconomicwellbeingisgenerallydetermined
WorldCup. by theamountofgoodsandservicesthatwe
C. Wehaveonechanceinamillionfor winningthe consume.
WorldCup. C. Oureconomicwellbeingisgenerallydetermined
through theamountofgoodsandservicesthat
weconsume.

GMATClub contributingtoeachotherslearning VERBAL GMATClub contributingtoeachotherslearning VERBAL

Idioms Idioms
166 168

Correctanswer: Correctanswer:
A. Wehaveonechanceinamillionof winningthe A. Oureconomicwellbeingisgenerallydetermined
WorldCup. from theamountofgoodsandservicesthatwe
consume.
B. Wehaveoneinamillionchancesto winthe
B. Oureconomicwellbeingisgenerallydetermined
WorldCup. by theamountofgoodsandservicesthatwe
C. Wehaveonechanceinamillionfor winningthe consume.
WorldCup. C. Oureconomicwellbeingisgenerallydetermined
through theamountofgoodsandservicesthat
weconsume.

GMATClub contributingtoeachotherslearning VERBAL GMATClub contributingtoeachotherslearning VERBAL

GMAT Club 2011 109


Idioms Idioms
169 171

Whichsentenceisidiomaticallycorrect? Whichsentenceisidiomaticallycorrect?
A. Sheiscreditedfor solvingnumerouscases. A. Whyhasn'tCongressmandatedthat thetotal
premiumcostbe shownaswagesoneverypay
B. Sheiscreditedwith solvingnumerouscases. stubbutnottaxed?
C. Sheiscreditedasbeing agreatdetective. B. Whyhasn'tCongressamandatefor thetotal
premiumcosttobe shownaswagesoneverypay
stubbutnottaxed?
C. Whyhasn'tCongressmandatedthat thetotal
premiumcostare shownaswagesoneverypay
stubbutnottaxed?

GMATClub contributingtoeachotherslearning VERBAL GMATClub contributingtoeachotherslearning VERBAL

Idioms Idioms
170 172

Correctanswer: Correctanswer:
A. Sheiscreditedfor solvingnumerouscases. A. Whyhasn'tCongressmandatedthat thetotal
premiumcostbe shownaswagesoneverypay
B. Sheiscreditedwith solvingnumerouscases. stubbutnottaxed?
C. Sheiscreditedasbeing agreatdetective. B. Whyhasn'tCongressamandatefor thetotal
premiumcosttobe shownaswagesoneverypay
stubbutnottaxed?
C. Whyhasn'tCongressmandatedthat thetotal
premiumcostare shownaswagesoneverypay
stubbutnottaxed?

GMATClub contributingtoeachotherslearning VERBAL GMATClub contributingtoeachotherslearning VERBAL

GMAT Club 2011 110


Idioms Idioms
173 175

Whichsentenceisidiomaticallycorrect? Whichsentenceisidiomaticallycorrect?
A. A13yearoldgirlwasmistakenlyabductedinMarch A. Butbothconsider theproblemsas necessary
2008bytraffickerswhowerebelievedtohave inconveniencestofeaturetheirconferencesand
mistaken heras thenieceofasuspecteddrugdealer. programs.
B. A13yearoldgirlwasmistakenlyabductedinMarch B. Butbothconsider theproblemstobe necessary
2008bytraffickerswhowerebelievedtohave inconveniencestofeaturetheirconferencesand
mistaken herfor thenieceofasuspecteddrug
programs.
dealer.
C. A13yearoldgirlwasmistakenlyabductedinMarch C. Butbothconsider theproblemsnecessary
2008bytraffickerswhowerebelievedtohave inconveniencestofeaturetheirconferencesand
mistaken herto thenieceofasuspecteddrugdealer. programs.

GMATClub contributingtoeachotherslearning VERBAL GMATClub contributingtoeachotherslearning VERBAL

Idioms Idioms
174 176

Correctanswer: Correctanswer:
A. A13yearoldgirlwasmistakenlyabductedinMarch A. Butbothconsider theproblemsas necessary
2008bytraffickerswhowerebelievedtohave inconveniencestofeaturetheirconferencesand
mistaken heras thenieceofasuspecteddrugdealer. programs.
B. A13yearoldgirlwasmistakenlyabductedinMarch B. Butbothconsider theproblemstobe necessary
2008bytraffickerswhowerebelievedtohave inconveniencestofeaturetheirconferencesand
mistaken herfor thenieceofasuspecteddrug
programs.
dealer.
C. A13yearoldgirlwasmistakenlyabductedinMarch C. Butbothconsider theproblemsnecessary
2008bytraffickerswhowerebelievedtohave inconveniencestofeaturetheirconferencesand
mistaken herto thenieceofasuspecteddrugdealer. programs.

GMATClub contributingtoeachotherslearning VERBAL GMATClub contributingtoeachotherslearning VERBAL

GMAT Club 2011 111


Idioms Idioms
177 179

Whichsentenceisidiomaticallycorrect? Whichsentenceisidiomaticallycorrect?
A. Thedrugmakerproposed thelitigationbe A. Theastronautswouldprobablybestusetheir
centralizedintheUSDistrictCourtforthe remainingtimelivingandworkingonMars
NorthernDistrictofOhio. insteadof dyingathome.
B. Thedrugmakerproposedthat thelitigationisto B. Theastronautswouldprobablybestusetheir
be centralizedintheUSDistrictCourtforthe remainingtimelivingandworkingonMarsrather
NorthernDistrictofOhio. dyingathome.
C. Thedrugmakerproposedthat thelitigationbe C. Theastronautswouldprobablybestusetheir
centralizedintheUSDistrictCourtforthe remainingtimelivingandworkingonMarsrather
NorthernDistrictofOhio. than dyingathome.

GMATClub contributingtoeachotherslearning VERBAL GMATClub contributingtoeachotherslearning VERBAL

Idioms Idioms
178 180

Correctanswer: Correctanswer:
A. Thedrugmakerproposed thelitigationbe A. Theastronautswouldprobablybestusetheir
centralizedintheUSDistrictCourtforthe remainingtimelivingandworkingonMars
NorthernDistrictofOhio. insteadof dyingathome.
B. Thedrugmakerproposedthat thelitigationisto B. Theastronautswouldprobablybestusetheir
be centralizedintheUSDistrictCourtforthe remainingtimelivingandworkingonMarsrather
NorthernDistrictofOhio. dyingathome.
C. Thedrugmakerproposedthat thelitigationbe C. Theastronautswouldprobablybestusetheir
centralizedintheUSDistrictCourtforthe remainingtimelivingandworkingonMarsrather
NorthernDistrictofOhio. than dyingathome.

GMATClub contributingtoeachotherslearning VERBAL GMATClub contributingtoeachotherslearning VERBAL

GMAT Club 2011 112


Idioms Idioms
181 183

Whichsentenceisidiomaticallycorrect? Whichsentenceisidiomaticallycorrect?
A. Thecompanyrequires himhold stockvaluedat A. Cubanpovertyisaresultof theAmericantrade
$4.4million,fourtimeshisannualbasesalaryof embargo.
$1.1million.
B. Theresultof theAmericanembargowas Cuba
B. Thecompanyrequires himtohold stockvalued
at$4.4million,fourtimeshisannualbasesalary impoverished.
of$1.1million. C. Resultingfrom theAmericanembargo,Cuba
C. Thecompanyrequires himholding stockvalued impoverished.
at$4.4million,fourtimeshisannualbasesalary
of$1.1million.

GMATClub contributingtoeachotherslearning VERBAL GMATClub contributingtoeachotherslearning VERBAL

Idioms Idioms
182 184

Correctanswer: Correctanswer:
A. Thecompanyrequires himhold stockvaluedat A. Cubanpovertyisaresultof theAmericantrade
$4.4million,fourtimeshisannualbasesalaryof embargo.
$1.1million.
B. Theresultof theAmericanembargowas Cuba
B. Thecompanyrequires himtohold stockvalued
at$4.4million,fourtimeshisannualbasesalary impoverished.
of$1.1million. C. Resultingfrom theAmericanembargo,Cuba
C. Thecompanyrequires himholding stockvalued impoverished.
at$4.4million,fourtimeshisannualbasesalary
of$1.1million.

GMATClub contributingtoeachotherslearning VERBAL GMATClub contributingtoeachotherslearning VERBAL

GMAT Club 2011 113


Idioms Idioms
185 187

Whichsentenceisidiomaticallycorrect? Whichsentenceisidiomaticallycorrect?
A. Arisingof electricitypricesfuelledthe18percent A. Itseemslike thecompanyis slowlyrunningout
yearonyearjumpinthecompany'srevenuesto ofnewfeaturestoadd.
3.6billionzlotys.
B. Thecompanyseemsasifitis slowlyrunningout
B. Arisein electricitypricesfuelledthe18percent
yearonyearjumpinthecompany'srevenuesto ofnewfeaturestoadd.
3.6billionzlotys. C. Itseemsasif thecompanyis slowlyrunningout
C. Araisein electricitypricesfuelledthe18percent ofnewfeaturestoadd.
yearonyearjumpinthecompany'srevenuesto
3.6billionzlotys.

GMATClub contributingtoeachotherslearning VERBAL GMATClub contributingtoeachotherslearning VERBAL

Idioms Idioms
186 188

Correctanswer: Correctanswer:
A. Arisingof electricitypricesfuelledthe18percent A. Itseemslike thecompanyis slowlyrunningout
yearonyearjumpinthecompany'srevenuesto ofnewfeaturestoadd.
3.6billionzlotys.
B. Thecompanyseemsasifitis slowlyrunningout
B. Arisein electricitypricesfuelledthe18percent
yearonyearjumpinthecompany'srevenuesto ofnewfeaturestoadd.
3.6billionzlotys. C. Itseemsasif thecompanyis slowlyrunningout
C. Araisein electricitypricesfuelledthe18percent ofnewfeaturestoadd.
yearonyearjumpinthecompany'srevenuesto
3.6billionzlotys.

GMATClub contributingtoeachotherslearning VERBAL GMATClub contributingtoeachotherslearning VERBAL

GMAT Club 2011 114


Idioms Idioms
189 191

Whichsentenceisidiomaticallycorrect? Whichsentenceisidiomaticallycorrect?
A. Sweetpotatoesarenativein CentralAmericaand A. Everyproduct usesimilar recycledpapers,

Peru. cardboardandothermaterials,inadditionto
leadfreepaints,nontoxicgluesandinks.
B. Sweetpotatoesarenativefrom CentralAmerica
B. Allproducts usesimilar recycledpapers,
andPeru. cardboardandothermaterials,inadditionto
C. Sweetpotatoesarenativeto CentralAmericaand leadfreepaints,nontoxicgluesandinks.
Peru. C. Eachproduct usesimilar recycledpapers,
cardboardandothermaterials,inadditionto
leadfreepaints,nontoxicgluesandinks.

GMATClub contributingtoeachotherslearning VERBAL GMATClub contributingtoeachotherslearning VERBAL

Idioms Idioms
190 192

Correctanswer: Correctanswer:
A. Sweetpotatoesarenativein CentralAmericaand A. Everyproduct usesimilar recycledpapers,

Peru. cardboardandothermaterials,inadditionto
leadfreepaints,nontoxicgluesandinks.
B. Sweetpotatoesarenativefrom CentralAmerica
B. Allproducts usesimilar recycledpapers,
andPeru. cardboardandothermaterials,inadditionto
C. Sweetpotatoesarenativeto CentralAmericaand leadfreepaints,nontoxicgluesandinks.
Peru. C. Eachproduct usesimilar recycledpapers,
cardboardandothermaterials,inadditionto
leadfreepaints,nontoxicgluesandinks.

GMATClub contributingtoeachotherslearning VERBAL GMATClub contributingtoeachotherslearning VERBAL

GMAT Club 2011 115


Idioms Idioms
193 195

Whichsentenceisidiomaticallycorrect? Whichsentenceisidiomaticallycorrect?
A. Theleadinfrom'DancingWiththeStars'willbe A. Theaveragepricehasrisenatleast1centper
high,andsotoo theexpectations. gallonperdayforeightconsecutivedaysto
$2.952,10centsasmuchas aweekago.
B. Theleadinfrom'DancingWiththeStars'willbe
B. Theaveragepricehasrisenatleast1centper
high,andalso theexpectations. gallonperdayforeightconsecutivedaysto
C. Theleadinfrom'DancingWiththeStars'willbe $2.952,10centsmorethan thatofaweekago.
high,andsotoowillbe theexpectations. C. Theaveragepricehasrisenatleast1centper
gallonperdayforeightconsecutivedaysto
$2.952,10centsmorefrom aweekago.

GMATClub contributingtoeachotherslearning VERBAL GMATClub contributingtoeachotherslearning VERBAL

Idioms Idioms
194 196

Correctanswer: Correctanswer:
A. Theleadinfrom'DancingWiththeStars'willbe A. Theaveragepricehasrisenatleast1centper
high,andsotoo theexpectations. gallonperdayforeightconsecutivedaysto
$2.952,10centsasmuchas aweekago.
B. Theleadinfrom'DancingWiththeStars'willbe
B. Theaveragepricehasrisenatleast1centper
high,andalso theexpectations. gallonperdayforeightconsecutivedaysto
C. Theleadinfrom'DancingWiththeStars'willbe $2.952,10centsmorethan thatofaweekago.
high,andsotoowillbe theexpectations. C. Theaveragepricehasrisenatleast1centper
gallonperdayforeightconsecutivedaysto
$2.952,10centsmorefrom aweekago.

GMATClub contributingtoeachotherslearning VERBAL GMATClub contributingtoeachotherslearning VERBAL

GMAT Club 2011 116


Idioms Idioms
197 199

Whichsentenceisidiomaticallycorrect? Whichsentenceisidiomaticallycorrect?
A. SponsorsincreasinglyusetheInternetlikea A. Hazhadawayofspeaking thatwaslesslike
directmediachanneltocustomers. givinginformationandmorelikewarningsand
B. SponsorsincreasinglyusetheInternettobea omens.
directmediachanneltocustomers. B. Hazhadawayforspeaking thatwaslesslike
C. SponsorsincreasinglyusetheInternetasa direct givinginformationandmorelikewarningsand
mediachanneltocustomers. omens.
C. Hazhadawaytospeak thatwaslesslikegiving
informationandmorelikewarningsandomens.

GMATClub contributingtoeachotherslearning VERBAL GMATClub contributingtoeachotherslearning VERBAL

Idioms Idioms
198 200

Correctanswer: Correctanswer:
A. SponsorsincreasinglyusetheInternetlikea A. Hazhadawayofspeaking thatwaslesslike
directmediachanneltocustomers. givinginformationandmorelikewarningsand
B. SponsorsincreasinglyusetheInternettobea omens.
directmediachanneltocustomers. B. Hazhadawayforspeaking thatwaslesslike
C. SponsorsincreasinglyusetheInternetasa direct givinginformationandmorelikewarningsand
mediachanneltocustomers. omens.
C. Hazhadawaytospeak thatwaslesslikegiving
informationandmorelikewarningsandomens.

GMATClub contributingtoeachotherslearning VERBAL GMATClub contributingtoeachotherslearning VERBAL

GMAT Club 2011 117


Recommended Verbal GMAT Books

Ultimate Verbal Sets


MGMAT Guides + Verbal OG 2nd ed
Veritas Prep Guides + Verbal OG 2nd ed

All-in-one Verbal Books


KaplanVerbal Workbook
PowerScore Verbal Bible

GMAT book reviews: http://gmatclub.com/books

GMAT Club 2011 118

GMAT Club contributing to each others learning


Best GMAT Courses
Manhattan GMAT Course - $100 discount
Classroom courses in many cities and online
http://gmatclub.com/mgmat
Princeton Review GMAT Club $150 discount
Classroom and online courses from $499
http://gmatclub.com/princeton
Veritas Prep 10% discount all products!
Save up to $650 on Veritas Prep GMAT courses as well
as Admissions Consulting Packages
http://gmatclub.com/veritas
GMAT Club 2011 119

GMAT Club contributing to each others learning


About GMAT Flashcards
Thank you for downloading GMAT Clubs GMAT
Flashcards. These flashcards have been put together
by a number of GMAT Club members ( Walker,
Bunuel, Shrouded1, Dzyubam, Whiplash2411, and
others)
Please visit http://gmatclub.com/flashcards
for the latest version of the GMAT Flashcards
If you have any suggestions how to improve these
flashcards, please visit the same page to leave your
feedback: http://gmatclub.com/flashcards
GMAT Club 2011 120

GMAT Club contributing to each others learning

You might also like